Você está na página 1de 114

Cálculo II - Módulo I

Profa . Lisiane Ramires Meneses

Prof. Odair Noskoski

Pelotas, RS.

Março de 2021
Conteúdo

1 Funções de Várias Variáveis Reais 1


1.1 Funções de Duas Variáveis Reais . . . . . . . . . . . . . . . . . . . . . . . . 1
1.1.1 Domı́nio e Imagem . . . . . . . . . . . . . . . . . . . . . . . . . . . 2
1.1.2 Gráfico de Funções de Duas Variáveis . . . . . . . . . . . . . . . . . 6
1.1.3 Curvas de Nı́vel . . . . . . . . . . . . . . . . . . . . . . . . . . . . . 7
1.2 Funções de Três Variáveis Reais . . . . . . . . . . . . . . . . . . . . . . . . 12
1.2.1 Domı́nio e Imagem . . . . . . . . . . . . . . . . . . . . . . . . . . . 12
1.2.2 Superfı́cies de Nı́vel . . . . . . . . . . . . . . . . . . . . . . . . . . . 13
1.3 Funções de “n” Variáveis Reais . . . . . . . . . . . . . . . . . . . . . . . . 14

2 Limite e Continuidade 15
2.1 Limite . . . . . . . . . . . . . . . . . . . . . . . . . . . . . . . . . . . . . . 15
2.2 Continuidade . . . . . . . . . . . . . . . . . . . . . . . . . . . . . . . . . . 19

3 Derivadas Parciais 21
3.1 Derivadas Parciais de Funções de Duas Variáveis . . . . . . . . . . . . . . . 21
3.1.1 Regra Prática para Determinar Derivadas Parciais . . . . . . . . . . 22
3.2 Interpretação Geométrica . . . . . . . . . . . . . . . . . . . . . . . . . . . 26
3.3 Diferenciação Parcial Implı́cita . . . . . . . . . . . . . . . . . . . . . . . . . 27
3.4 Derivadas Parciais de Segunda Ordem . . . . . . . . . . . . . . . . . . . . 30
3.4.1 Aplicações . . . . . . . . . . . . . . . . . . . . . . . . . . . . . . . . 32
3.5 Derivadas Parciais de Ordem Superior . . . . . . . . . . . . . . . . . . . . 33
3.6 Diferenciabilidade . . . . . . . . . . . . . . . . . . . . . . . . . . . . . . . . 33
3.7 Regra da Cadeia . . . . . . . . . . . . . . . . . . . . . . . . . . . . . . . . 35
3.8 Derivadas Direcionais e o Vetor Gradiente . . . . . . . . . . . . . . . . . . 42
3.8.1 Derivadas Direcionais . . . . . . . . . . . . . . . . . . . . . . . . . . 42
3.9 Vetor Gradiente . . . . . . . . . . . . . . . . . . . . . . . . . . . . . . . . . 46
3.9.1 Propriedades Algébricas dos Gradientes . . . . . . . . . . . . . . . . 47
3.10 Valores Extremos de Funções de Duas Variáveis . . . . . . . . . . . . . . . 50
3.10.1 Extremos Absolutos em Conjuntos Fechados e Limitados . . . . . . 55
3.10.2 Problemas Aplicados . . . . . . . . . . . . . . . . . . . . . . . . . . 59
3.11 Multiplicadores de Lagrange . . . . . . . . . . . . . . . . . . . . . . . . . . 62

4 Integrais Múltiplas 69
4.1 Integrais Duplas - Conceitos Preliminares . . . . . . . . . . . . . . . . . . . 69
4.2 Problema Motivador . . . . . . . . . . . . . . . . . . . . . . . . . . . . . . 69
4.3 Funções Integráveis . . . . . . . . . . . . . . . . . . . . . . . . . . . . . . . 71
4.4 Propriedades da Integral . . . . . . . . . . . . . . . . . . . . . . . . . . . . 71
4.5 Integrais Iteradas - Teorema de Fubini . . . . . . . . . . . . . . . . . . . . 72
4.6 Integrais duplas sobre regiões não retangulares limitadas . . . . . . . . . . 74
4.7 Integrais Duplas em Coordenadas Polares . . . . . . . . . . . . . . . . . . . 79
4.8 Área de Superfı́cie . . . . . . . . . . . . . . . . . . . . . . . . . . . . . . . 84
4.9 Integrais Triplas - Conceitos Preliminares . . . . . . . . . . . . . . . . . . . 86
4.10 Funções Integráveis . . . . . . . . . . . . . . . . . . . . . . . . . . . . . . . 88
4.11 Mudança de Variáveis em Integrais Múltiplas . . . . . . . . . . . . . . . . . 92
4.11.1 Integral Dupla . . . . . . . . . . . . . . . . . . . . . . . . . . . . . . 92
4.11.2 Integral Tripla . . . . . . . . . . . . . . . . . . . . . . . . . . . . . . 101
4.11.3 Coordenadas Cilı́ndricas e Esféricas . . . . . . . . . . . . . . . . . . 103

5 Funções Vetoriais 110


5.1 Introdução . . . . . . . . . . . . . . . . . . . . . . . . . . . . . . . . . . . 110
5.2 Gráfico de Funções Vetoriais . . . . . . . . . . . . . . . . . . . . . . . . . 111
5.3 Limite e Continuidade de Funções Vetoriais . . . . . . . . . . . . . . . . . 113
5.4 Derivadas de Funções Vetoriais . . . . . . . . . . . . . . . . . . . . . . . . 114
5.4.1 Regras de Diferenciação . . . . . . . . . . . . . . . . . . . . . . . . 115
5.4.2 Interpretação geométrica de r′ . . . . . . . . . . . . . . . . . . . . . 116
5.4.3 Planos Tangentes . . . . . . . . . . . . . . . . . . . . . . . . . . . . 117
5.4.4 Reta Normal . . . . . . . . . . . . . . . . . . . . . . . . . . . . . . 118
5.5 Integrais de Funções Vetoriais . . . . . . . . . . . . . . . . . . . . . . . . . 120
5.5.1 Antiderivada de Funções Vetoriais . . . . . . . . . . . . . . . . . . . 121
5.6 Comprimento de Arco . . . . . . . . . . . . . . . . . . . . . . . . . . . . . 121
5.7 Comprimento de Arco . . . . . . . . . . . . . . . . . . . . . . . . . . . . . 122
5.7.1 Regras de Integração . . . . . . . . . . . . . . . . . . . . . . . . . . 124
5.8 Parametrizações e Comprimento de Arco . . . . . . . . . . . . . . . . . . . 124

6 Cálculo Vetorial 125


6.1 Continuidade de Campos Vetoriais . . . . . . . . . . . . . . . . . . . . . . 126
6.2 Campos Gradientes . . . . . . . . . . . . . . . . . . . . . . . . . . . . . . . 127
6.3 Campos Vetoriais Conservativos e Funções Potenciais . . . . . . . . . . . . 128
6.4 Rotacional e Divergente . . . . . . . . . . . . . . . . . . . . . . . . . . . . 129
6.4.1 Rotacional . . . . . . . . . . . . . . . . . . . . . . . . . . . . . . . . 129
6.4.2 Divergente . . . . . . . . . . . . . . . . . . . . . . . . . . . . . . . . 131
6.4.3 Propriedades do Rotacional e Divergente . . . . . . . . . . . . . . . 132
6.5 Integrais de Linha . . . . . . . . . . . . . . . . . . . . . . . . . . . . . . . . 132
Capı́tulo 1

Funções de Várias Variáveis Reais

No estudo de fenômenos fı́sicos, uma quantidade normalmente depende de duas ou mais


variáveis. Portanto, precisamos ampliar a idéia básica do cálculo de funções de uma única
variável para funções de várias variáveis.

A seguir são apresentados alguns exemplos de funções que dependem de mais de uma
variável.

• A lei dos gases ideais P V = nRT , onde n e R são constantes, permite expressar
qualquer uma das variáveis P , V e T como funções das outras duas.

• A quantidade de energia utilizável que um painel solar pode captar depende de sua
eficiência, do seu ângulo de inclinação um relação aos raios solares, do ângulo de
elevação do sol acima do horizonte, e outros fatores.

• O volume de um cilindro circular reto depende de seu raio r e de sua altura h. De


fato, sabemos que V = πr2 h. Podemos dizer que V é função de r e h, e escrevemos
V (r, h) = πr2 h

1.1 Funções de Duas Variáveis Reais


Definição 1 Uma função f de duas variáveis reais x e y, é uma lei que associa cada
ponto (x, y) de algum subconjunto D do R2 a um único número real denotado por
z = f (x, y).

Quando escrevemos z = f (x, y), queremos tornar explı́citos os valores tomados por f em
um ponto genérico (x, y) ∈ D. As variáveis x e y são variáveis independentes, e z é a
variável dependene.
2 Meneses, L. R.; Noskoski, O. Cálculo II

1.1.1 Domı́nio e Imagem


Definição 2 Seja f uma função de duas variáveis reais x e y, com z = f (x, y), defini-
mos o domı́nio de f , denotado por D(f ), como o maior conjunto do R2 para o qual a
lei de formação de f gera números reais a menos que esse domı́nio seja especificado de
forma explı́cita.

Definição 3 A imagem de uma função f de duas variáveis reais, denotada por Im(f ),
é definida como o conjunto dos valores z = f (x, y), com (x, y) ∈ D.
y

D (x1 , y1 )

x f (x2 , y2 ) 0 f (x1 , y1 ) z
(x2 , y2 )

A seguir apresentamos alguns exemplos de obtenção do domı́nio.

Exemplo 1 Determine o domı́nio de f , sendo f definida por:



x+y+1
f (x, y) = .
x−1
Solução: A expressão para f está bem definida se o denominador for diferente de zero e
o número cuja raiz quadrada será extraı́da for não negativo. Portanto, o domı́nio de f é:
D(f ) = {(x, y) ∈ R2 /x + y + 1 ≥ 0 e x 6= 1}.
A desigualdade x + y + 1 ≥ 0, ou y ≥ −x − 1, descreve os pontos que estão sobre ou acima
da reta de equação y = −x − 1, ao passo que x 6= 1 significa que os pontos sobre a reta
x = 1 precisam ser excluı́dos do domı́nio. A figura a seguir, mostra o esboço gráfico do
domı́nio da função f , definida acima.

y = −x − 1 x=1

−1 x
−1
3 Meneses, L. R.; Noskoski, O. Cálculo II

Exemplo 2 Determine o domı́nio de f , sendo f definida por f (x, y) = x ln(y 2 − x).

Solução: Como ln(y 2 − x) é definido somente quando y 2 − x > 0, ou seja, x < y 2 , segue
que o domı́nio de f será:
D(f ) = {(x, y) ∈ R2 /x < y 2 }.

Isso representa o conjunto de pontos à esquerda da parábola x = y 2 . O esboço gráfico do


domı́nio é apresentado na figura a seguir.

−1 0 1 2 3 x
−1

−2

Exemplo 3 Determine o domı́nio de f , sendo f dada por:


p p
f (x, y) = y − x2 + 2x − y.

Solução: Esta função está definida se o número cuja raiz quadrada será extraı́da for não
negativo. Assim, o domı́nio de f é

D(f ) = {(x, y) ∈ R2 /y ≥ x2 e y ≤ 2x}.

A desigualdade y ≥ x2 descreve os pontos que estão acima da parábola de equação y = x2 ,


enquanto que y ≤ 2x representa o conjunto dos pontos que estão abaixo da reta de equação
y = 2x. Assim, o esboço gráfico do domı́nio é apresentado na figura a seguir.
4 Meneses, L. R.; Noskoski, O. Cálculo II

Exemplo 4 Determine o domı́nio da função z = f (x, y) definida dada por

z 2 + 4 = x2 + y 2 , z ≥ 0.

Solução: Como z ≥ 0, a expressão z 2 + 4 = x2 + y 2 pode ser reescrita como z =


p
x2 + y 2 − 4. Note que, esta função está definida se x2 + y 2 − 4 ≥ 0. Assim, o domı́nio
de f é dado por:
D(f ) = {(x, y) ∈ R2 /x2 + y 2 ≥ 4}.

A desigualdade x2 +y 2 ≥ 4 descreve os pontos que estão sobre a circunferência de equação


x2 + y 2 = 4 e os pontos exteriores a ela, como podemos ver na ilustração a seguir.

y
2

1.

−2 −1. 0 1. 2 x
−1.

−2

Exemplo 5 Determine o domı́nio da função f definida dada por


r
x+y
f (x, y) = .
x−y
x+y
Solução: Esta função está definida se ≥ 0. Note que, uma fração assume um valor
x−y
positivo, se o numerador e o denominador forem ambos positivos, ou, se o numerador e o
denominador forem ambos negativos. Assim, temos:

x+y  x+y ≥0 e x−y >0

≥0 ⇒ ou
x−y 

x+y ≤0 e x−y <0

Logo, o domı́nio da função f é:

D(f ) = {(x, y) ∈ R2 /y ≥ −x e y < x ou y ≤ −x e y > x}.

As desigualdades y ≥ −x e y < x descrevem os pontos que estão acima da reta de equação


y = −x e abaixo da reta de equação y = x, enquanto que as desigualdades y ≤ −x e
y > x descrevem os pontos que estão abaixo da reta de equação y = −x e acima da reta
de equação y = x. A figura a seguir apresenta o esboço gráfico da função f .
5 Meneses, L. R.; Noskoski, O. Cálculo II

Exemplo 6 Determine o domı́nio da função f definida dada por

f (x, y) = arcsin(xy).

Solução: Esta função está definida se −1 ≤ xy ≤ 1. Assim, o domı́nio de f é dado por:

D(f ) = {(x, y) ∈ R2 / − 1 ≤ xy ≤ 1}.

A desigualdade xy ≥ −1 descreve os pontos que estão sobre a hipérbole de equação


xy = −1, bem como os pontos interiores aos ramos dela; e a desigualdade xy ≤ 1 descreve
os pontos que estão sobre a hipérbole de equação xy = 1, bem como os pontos interiores
aos ramos dela. Assim, o esboço gráfico do domı́nio da função definida acima é dado pela
intersecção das regiões mencionadas, como podemos ver na ilustração a seguir.
6 Meneses, L. R.; Noskoski, O. Cálculo II

1.1.2 Gráfico de Funções de Duas Variáveis


Definição 4 Se f é uma função de duas variáveis com domı́nio D, então o gráfico de
f é o conjunto de todos os pontos (x, y, z) em R3 tal que z = f (x, y) e (x, y) pertençam
a D.

Considerando-se um sistema ortogonal de coordenadas cartesianas no R3 , o gráfico de


f pode ser pensado como o lugar geométrico descrito pelo ponto (x, y, f (x, y)), quando
(x, y) percorre o domı́nio de f . Assim, o gráfico de uma função com duas variáveis é uma
superfı́cie S com equação z = f (x, y).

Vejamos alguns exemplos de esboços gráficos.

Exemplo 7 O gráfico da função constante f (x, y) = k é um plano paralelo ao plano xy.

Exemplo 8 O gráfico da função definida por z = 2x+y é um plano passando pela origem
e normal ao vetor →

n = (2, 1, −1).
Este plano, cujo esboço gráfico é apresentado a seguir, é determinado pelas retas de
equações:
( (
x=0 y=0
r1 : e r2 :
z=y z = 2x
7 Meneses, L. R.; Noskoski, O. Cálculo II

p p
Exemplo 9 O gráfico de f (x, y) = 16 − x2 − y 2 é o gráfico da equação z = 16 − x2 − y 2 .
Note que, após elevar ambos os membros ao quadrado e realizar algumas manipulações
algébricas, a equação anterior pode ser reescrita como

x2 + y 2 + z 2 = 16,

a qual representa uma esfera de raio 4, centrada na origem. Como z ≥ 0, o gráfico é


somente a semi-esfera superior.

1.1.3 Curvas de Nı́vel

Definição 5 As curvas de nı́vel ou curvas de contorno de uma função f de duas


variáveis são aquelas com equação f (x, y) = k, onde k é uma constante real.

Se a superfı́cie z = f (x, y) for interceptada pelo plano horizontal z = k, então as curvas


de nı́vel f (x, y) = k são apenas traços do gráfico de f no plano horizontal z = k projetado
sobre o plano xy.

A figura a seguir ilustra este fato.


8 Meneses, L. R.; Noskoski, O. Cálculo II

Um conjunto de curvas de nı́vel para z = f (x, y) é chamado de mapa de contorno de f .

Um exemplo comum de curvas de nı́vel ocorre em mapas topográficos de regiões monta-


nhosas. As curvas de nı́vel são aquelas em que a elevação em relação ao nı́vel do mar
é constante. A superfı́cie será mais inclinada onde as curvas de nı́vel estiverem mais
próximas umas das outras. Ela é mais ou menos plana onde as curvas de nı́vel estão
distantes umas das outras. Isto pode ser observado na ilustração abaixo.

Outro exemplo comum é a função pressão p(x, y) definida nos pontos geográficos (x, y),
representados no mapa. Uma curva conectando os pontos de pressão atmosférica constante
sobre um mapa meteorológico é chamada de linha isobárica ou isóbara. Matematicamente,
as isóbaras são curvas de nı́vel para a função pressão. Linhas isobáricas muito próximas
correspodem a inclinações ı́ngremes no gráfico da função pressão, e estão usualmente
associados a fortes ventos, quanto maior a inclinação, maior será a velocidade do vento.
9 Meneses, L. R.; Noskoski, O. Cálculo II

Exemplo 10 O gráfico da função f (x, y) = y 2 − x2 no R3 é um parabolóide hiperbólico.

As curvas de nı́vel desta função tem a forma y 2 − x2 = k. Para k > 0, essas curvas são
hipérboles com eixo real sobre o eixo dos y; para k < 0, elas são hipérboles com eixo real
sobre o eixo dos x, e para k = 0, a curva de nı́vel consiste nas retas y + x = 0 e y − x = 0.
10 Meneses, L. R.; Noskoski, O. Cálculo II

Exemplo 11 O gráfico da superfı́cie z = 1 − 2x − y é o plano que passa pelos pontos


A( 12 , 0, 0), B(0, 1, 0) e C(0, 0, 1), ilustrado na figura a seguir.

A curva de nı́vel de altura k tem a equação 1 − 2x − y = k, a qual podemos reescrever


como
y = −2x + (1 − k).

Isto representa no plano xy, uma famı́lia de retas paralelas de inclinação −2. O mapa de
contorno é apresentado na figura a seguir.

A representação geométrica do gráfico de uma função de duas variáveis nem sempre é


tarefa fácil. Assim, quando se pretende ter uma visão geométrica da função, lança-se mão
de suas curvas de nı́vel, cuja representação geométrica é sempre mais fácil de ser obtida
do que o gráfico da função. Veja o exemplo a seguir.

Exemplo 12 Seja f a função definida por


1
f (x, y) = .
x2 + y 2
(a) Determine o domı́nio e a imagem de f .
(b) Desenhe as curvas de nı́vel de f .
(c) Esboce o gráfico de f .
11 Meneses, L. R.; Noskoski, O. Cálculo II

1
Solução: (a) Esta função não está definida na origem, pois 02 + 02 = 0 e 0
não existe.
Assim,
D(f ) = {(x, y) ∈ R2 /(x, y) 6= (0, 0)}
1
Como ≥ 0, temos
x2 + y2
Im(f ) = {z ∈ R/z > 0}.

(b) A curva de nı́vel correspondente a z = k, com k > 0, é:


1 1
⇒ x2 + y 2 =
x2 + y2 k
1
Assim, as curvas de nı́vel são circunferências concêntricas de centro na origem e raio √ .
k
Note que, quando k tende a +∞, o raio tende a zero e quando k tende a zero, o raio tende
a +∞.

y
1
k= 4

k=1
k=4
1 2 x

(
x=0
(c) O traço desta superfı́cie no plano yz é a curva de equação , enquanto que,
z = y12
(
y=0
o traço desta superfı́cie no plano xz é a curva de equação . Para cada k > 0, o
z = x12
(
z=k
plano z = k intercepta o gráfico de f segundo a circunferência .
x2 + y 2 = k1
12 Meneses, L. R.; Noskoski, O. Cálculo II

Observação 1 Note que a denominação curva de nı́vel varia de acordo com o que
a função f representa. Se f é uma distribuição de temperatura, ou seja, f (x, y) é
a temperatura no ponto (x, y), as curvas de nı́vel denominam-se isotermas (pontos
de temperatura constante); se f é a energia potencial de um certo campo de forças
bidimensionais, as curvas de nı́vel denominam-se curvas equipotenciais, etc.

1.2 Funções de Três Variáveis Reais


Definição 6 Uma função f de três variáveis reais x, y e z, é uma lei que associa
cada ponto (x, y, z) de algum subconjunto D do R3 a um único número real denotado
por w = f (x, y, z). Chamamos x, y e z de variáveis independentes e w de variável
dependente.

p
Exemplo 13 Seja f (x, y, z) = x2 + y 2 + z 2 , determine f (3, 0, −4) e f (−1, −1, −2).
p √
Solução: f (3, 0, −4) = 32 + 02 + (−4)2 = 9 + 0 + 16 = 5
p √ √
f (−1, −1, −2) = (−1)2 + (−1)2 + (−2)2 = 1 + 1 + 4 = 6

Note que, o valor funcional de f , representa a distância do ponto de coordenadas x,


y e z à origem.

1.2.1 Domı́nio e Imagem

Definição 7 Seja f uma função de três variáveis reais x, y e z, com w = f (x, y, z),
definimos o domı́nio de f , denotado por D(f ), como o maior conjunto do R3 para o qual
a lei de formação de f gera números reais a menos que esse domı́nio seja especificado
de forma explı́cita.

Definição 8 A imagem de uma função f de três variáveis reais, denotada por Im(f ),
é definida como o conjunto dos valores w = f (x, y, z), com (x, y, z) ∈ D.
13 Meneses, L. R.; Noskoski, O. Cálculo II

Exemplo 14 Determine e faça o esboço gráfico do domı́nio da função definida por


f (x, y, z) = ln(16 − 4x2 − 4y 2 − z 2 ).

Solução: Como ln(16 − 4x2 − 4y 2 − z 2 ) é definido somente quando 16 − 4x2 − 4y 2 − z 2 > 0,


x2 y 2 z 2
ou seja, + + < 1, temos que, o domı́nio de f é:
4 4 16
 2

3 x y2 z2
D(f ) = (x, y, z) ∈ R / + + <1
4 4 16

Isso representa o conjunto de pontos interiores ao elipsóide de centro na origem, cujo


gráfico é apresentado a seguir.

Observação 2 Note que o gráfico de y = f (x) é uma curva no R2 e o gráfico de


z = f (x, y) é uma superfı́cie no R3 , logo o número de dimensões necessárias para esses
gráficos é o número de variáveis mais 1. Conseqı̈¿ 21 entemente, não há maneira de fazer
o gráfico de uma função de três variáveis, uma vez que a quarta dimensão é necessária.

1.2.2 Superfı́cies de Nı́vel


Definição 9 As superfı́cies de nı́vel de uma função f de três variáveis reais são aquelas
com equação f (x, y, z) = k, onde k é uma constante.

Assim, de acordo com esta definição, se um ponto (x, y, z) se move ao longo de uma
superfı́cie de nı́vel, o valor de f (x, y, z) permanece fixo.

Exemplo 15 Determine as superfı́cies de nı́vel da função

f (x, y, z) = x2 + y 2 + z 2 .

Solução: As superfı́cies de nı́vel são x2 +y 2 +z 2 = k, onde k ≥ 0. Elas formam uma famı́lia



de esferas concêntricas com raio k. Veja a figura a seguir. Então, quando (x, y, z) varia
sobre uma das esferas com centro O, o valor de f (x, y, z) permanece fixo.
14 Meneses, L. R.; Noskoski, O. Cálculo II

1.3 Funções de “n” Variáveis Reais


A definição de função de duas ou três variáveis pode ser estendida para mais variáveis,
conforme definição abaixo.

Definição 10 Uma função f de n variáveis reais x1 , x2 , x3 , ... xn é uma lei que associa
cada ponto (x1 , x2 , x3 , ..., xn ) de algum subconjunto D do espaço Rn a um único número
real, denotado por f (x1 , x2 , x3 , ..., xn ).
Capı́tulo 2

Limite e Continuidade

2.1 Limite
Definição 11 Seja f uma função de duas variáveis reais, definida em um domı́nio
D ⊂ R2 . Dizemos que f possui limite L quando (x, y) ∈ D aproxima-se (x0 , y0 ), se,
dado qualquer número positivo ǫ, existe um número positivo δ tal que, para todo (x, y)
no domı́nio de f ,
p
0< (x − x0 )2 + (y − y0 )2 < δ ⇒ |f (x, y) − L| < ǫ,

e escrevemos
lim f (x, y) = L.
(x,y)→(x0 ,y0 )

Considere a figura a seguir, onde D representa o domı́nio de f e (x, y) ∈ D. Dado


ǫ > 0, podemos achar δ > 0 tal que, se (x, y) 6= (x0 , y0 ), sua imagem estará entre os
planos horizontais z = L − ǫ e z = L + ǫ.

y
δ
(x, y)
D y0
f (x, y) z
x0 x L−ε L+ε

Para funções de uma variável real, quando fazemos x se aproximar de x0 , há apenas dois
sentidos possı́veis de aproximação: pela esquerda ou pela direita. Assim, definimos os
16 Meneses, L. R.; Noskoski, O. Cálculo II

limites laterais no ponto x0 , isto é

lim f (x) e lim f (x).


x→x−
0 x→x+
0

Lembre-se do Cálculo I que, se lim− f (x) 6= lim+ f (x), então lim f (x) não existe. Para
x→x0 x→x0 x→x0
funções de duas ou três variáveis, a situação é mais complicada, pois há infinitas maneiras
de (x, y) se aproximar de (x0 , y0 ) por uma quantidade infinita de direções, bastando que
(x, y) se mantenha no domı́nio de f .

Se o limite existe, f (x, y) deve se aproximar do mesmo valor limite, independentemente


do modo como (x, y) se aproxima de (x0 , y0 ). Assim, se acharmos dois caminhos dife-
rentes de aproximação ao longo dos quais f (x, y) tem limites diferentes, segue então que
lim f (x, y) não existe.
(x,y)→(x0 ,y0 )

x2 − y 2
Exemplo 16 Mostre que lim não existe.
(x,y)→(0,0) x2 + y 2

x2 − y 2
Solução: Considere f (x, y) = . Inicialmente determinaremos este limite ao longo
x2 + y 2
2
do eixo x. Assim, tomando y = 0, temos f (x, 0) = xx2 = 1 para todo x 6= 0, logo

f (x, y) → 1 quando (x, y) → (0, 0) ao longo do eixo x.

Agora, determinaremos este limite ao longo do eixo y, fazendo-se x = 0, logo f (0, y) =


2
− yy2 = −1, para todo y 6= 0. Assim,

f (x, y) → −1 quando (x, y) → (0, 0) ao longo do eixo y.

Como f tem dois limites diferentes ao longo de duas retas distintas, concluı́mos que o
limite não existe.
17 Meneses, L. R.; Noskoski, O. Cálculo II

x2 − y 2
Figura 2.2: Gráfico da função definida por f (x, y) =
x2 + y 2

Observação 3 Não podemos provar que f (x, y) → L quando (x, y) → (x0 , y0 )


provando que f (x, y) → L quando (x, y) → (x0 , y0 ) ao longo de uma curva especificada
ou mesmo de uma famı́lia de curvas. Isto porque, pode existir alguma curva fora da
famı́lia para a qual o limite não exista ou tenha um limite que é diferente de L.

2x2 y
Exemplo 17 Determine lim , se este existir.
(x,y)→(0,0) x2 + y 2

Solução: Inicilamente determinaremos este limite ao longo de uma reta qualquer que
passa pela origem. Tomando y = mx, temos
2x2 (mx) 2x3 m 2mx
f (x, y) = f (x, mx) = 2 2
= 2 2 2
=
x + (mx) x +x m 1 + m2
Portanto, f (x, y) → 0 quando (x, y) → (0, 0) ao londo de y = mx.

Fazendo-se (x, y) se aproximar de (x0 , y0 ) ao longo da parábola de equação y = x2 também


obtemos o limite 0. Assim, fazendo-se y = x2 , temos:
2x2 x2 2x4 2x2
f (x, y) = f (x, x2 ) = = =
x2 + x 4 x2 (1 + x2 ) 1 + x2
Logo, f (x, y) → 0 quando (x, y) → (0, 0) ao longo de y = x2 .

Isto não prova a existência do limite igual a 0, mas suspeitamos que o limite exista e
seja igual a 0.

Para provar a existência deste limite, devemos provar que dado ǫ > 0, existe um δ > 0,
tal que
2x2 y p

x2 + y 2 − 0 <ǫ
sempre que 0 < x2 + y 2 < δ,

ou seja
2x2 |y| p
<ǫ sempre que 0 < x2 + y 2 < δ.
x2 + y 2
18 Meneses, L. R.; Noskoski, O. Cálculo II

x2
Note que x2 ≤ x2 + y 2 , pois y 2 ≥ 0. Logo ≤ 1 e, portanto
x2 + y 2

2x2 |y| p
2 ≤ 2
p
≤ 2|y| = 2 y x2 + y 2 < 2δ.
x2 + y 2
p
Assim, se escolhermos δ = 2ǫ e considerando 0 < x2 + y 2 < δ, temos:

2x2 y p ǫ
2 2
x2 + y 2 − 0 ≤ 2 x + y < 2δ = 2 · 2 = ǫ.

Logo,
2x2 y
lim = 0,
(x,y)→(x0 ,y0 ) x2 + y 2

Provando que o limite existe.

2x2 y
Figura 2.3: Gráfico da função definida por f (x, y) =
x2 + y 2

Teorema 1 Sejam f e g funções de duas variáveis x e y, ambas definidas no domı́nio


D do plano xy. Sejam

lim f (x, y) = L1 e lim g(x, y) = L2 .


(x,y)→(x0 ,y0 ) (x,y)→(x0 ,y0 )

Então,
lim [f (x, y) + g(x, y)] = L1 + L2 (2.1)
(x,y)→(x0 ,y0 )

lim [f (x, y) · g(x, y)] = L1 · L2 (2.2)


(x,y)→(x0 ,y0 )
 
f (x, y) L1
lim = , com L2 6= 0 (2.3)
(x,y)→(x0 ,y0 ) g(x, y) L2
19 Meneses, L. R.; Noskoski, O. Cálculo II

Definição 12 Seja f uma função de três variáveis reais, definida em um domı́nio D.


Dizemos que f possui limite L quando (x, y, z) ∈ D aproxima-se (x0 , y0 , z0 ), se, dado
qualquer número positivo ǫ, existe um número positivo δ tal que, para todo (x, y, z) no
domı́nio de f ,
p
|f (x, y, z) − L| < ǫ sempre que 0 < (x − x0 )2 + (y − y0 )2 + (z − z0 )2 < δ

e escrevemos
lim f (x, y, z) = L.
(x,y,z)→(x0 ,y0 ,z0 )

2.2 Continuidade
Definição 13 Uma função f de duas variáveis é dita contı́nua em (x0 , y0 ) se

lim f (x, y) = f (x0 , y0 ).


(x,y)→(x0 ,y0 )

Dizemos que f é contı́nua se f for contı́nua em todo ponto (x0 , y0 ) de seu domı́nio.

Teorema 2 (a) Se g e h forem funções contı́nuas de uma variável real, então


f (x, y) = g(x) · h(y) é uma função contı́nua de x e y.
(b) Se g for uma função contı́nua de uma variável e h for uma função de duas variáveis
contı́nua, então sua composição f (x, y) = g(h(x, y)) é uma função contı́nua de x e y.

Observação 4 (a) A composição de funções contı́nuas é contı́nua.


(b) A soma, diferença ou produto de funções contı́nuas é contı́nuo.
(c) O quociente de funções contı́nuas é contı́nuo, exceto onde o denominador é zero.

x2 − y 2
Exemplo 18 Determine onde a função f , definida por f (x, y) = é contı́nua.
x2 + y 2
Solução: A função f , não está definida em (0, 0), logo é descontı́nua neste ponto. Como
trata-se de uma função racional, ela é contı́nua em seu domı́nio, o que corresponde a
D = {(x, y) ∈ R2 /(x, y) 6= (0, 0)}.

y
Exemplo 19 Determine onde a função h, definida por h(x, y) = arctan é contı́nua.
x
y
Solução: A função f (x, y) = é racional, assim contı́nua em todo R2 , exceto sobre a reta
x
x = 0. A função g(t) = arctan(t) é contı́nua. Logo, a função composta
y
g(f (x, y)) = arctan = h(x, y)
x
20 Meneses, L. R.; Noskoski, O. Cálculo II

é contı́nua, exceto em x = 0.

Definição 14 Dizemos que uma função f de três variáveis reais é contı́nua num dado
ponto (x0 , y0 , z0 ) se o limite da função e o valor da função forem o mesmo neste ponto,
isto é, se
lim f (x, y, z) = f (x0 , y0 , z0 ).
(x,y,z)→(x0 ,y0 ,z0 )
Capı́tulo 3

Derivadas Parciais

O volume V de um cilindro circular reto é dado pela fórmula V = πr2 h, onde r é o raio
da base do cilindro e h é a sua altura. Assim, podemos dizer que o volume é uma função
do raio e da altura. Poderı́amos nos perguntar: qual é a taxa de variação do volume se
o raio da base for mantido fixo e a altura for permitido variar ou se a altura for mantida
fixa e ao raio da base for permitido variar?
Estudaremos nesta seção as taxas de variação que envolvam duas ou mais variáveis.

3.1 Derivadas Parciais de Funções de Duas Variáveis


Definição 15 Seja f : Ω ⊂ R2 → R, uma função de duas variáveis x e y, e Ω um
∂f
aberto do R2 . Definimos a derivada parcial de f em relação a x e escrevemos (ou
∂x
fx (x, y)) como o limite

∂f f (x + ∆x, y) − f (x, y)
(x, y) = lim ,
∂x ∆x→0 ∆x
se este limite existir.

Esta derivada, em outras palavras, é obtida mantendo-se y fixo e fazendo-se x variar.

Definição 16 Seja f : Ω ⊂ R2 → R, uma função de duas variáveis x e y, e Ω um


∂f
aberto do R2 . Definimos a derivada parcial de f em relação a y e escrevemos (ou
∂y
fy (x, y)) como o limite

∂f f (x, y + ∆y) − f (x, y)


(x, y) = lim ,
∂y ∆y→0 ∆y
se este limite existir.

Esta derivada, em outras palavras, é obtida mantendo-se x fixo e fazendo-se y variar.


22 Meneses, L. R.; Noskoski, O. Cálculo II

A seguir apresentamos um exemplo do cálculo da derivada parcial utilizando-se a definição.

∂z
Exemplo 20 Sendo f uma função definida por z = f (x, y) = 3x2 y + y 2 , determine e
∂x
∂z
, utilizando a definição.
∂y

Solução:

∂z [3(x + ∆x)2 y + y 2 ] − (3x2 y + y 2 )


(a) = lim
∂x ∆x→0 ∆x
∂z 3x2 y + 6xy∆x + 3y∆x2 + y 2 − 3x2 y − y 2
= lim
∂x ∆x→0 ∆x
∂z ∆x(6xy + 3y∆x)
= lim
∂x ∆x→0 ∆x
∂z
= lim (6xy + 3y∆x)
∂x ∆x→0
∂z
= 6xy
∂x

∂z [3x2 (y + ∆y) + (y + ∆y)2 ] − (3x2 y + y 2 )


= lim
∂y ∆y→0 ∆y
∂z 3x2 y + 3x2 ∆y + y 2 + 2y∆y + ∆y 2 − 3x2 y − y 2
= lim
∂y ∆y→0 ∆y
∂z ∆y(3x2 + 2y + ∆y)
= lim
∂y ∆y→0 ∆y
∂z
= lim (3x2 + 2y + ∆y)
∂y ∆y→0
∂z
= 3x2 + 2y
∂y

3.1.1 Regra Prática para Determinar Derivadas Parciais


∂f
1. Para determinar , considere y como uma constante e diferencie f com relação
∂x
a x.
∂f
2. Para determinar , considere x como uma constante e diferencie f com relação
∂y
a y.
23 Meneses, L. R.; Noskoski, O. Cálculo II

∂f ∂f
Exemplo 21 Determine e , sendo z = f (x, y) = e2x sin y.
∂x ∂y

Solução:
∂f ∂ 2x ∂ ∂
= (e sin y) = e2x (sin y) + sin y (e2x ) = e2x · 0 + 2e2x sin y = 2e2x sin y
∂x ∂x ∂x ∂x
∂f ∂ 2x ∂ ∂
= (e sin y) = e2x (sin y) + sin y (e2x ) = e2x cos y + sin y · 0 = e2x cos y
∂y ∂y ∂y ∂y

Exemplo 22 Mostre que a função f , definida por


 xy
 , se (x, y) 6= (0, 0)
f (x, y) = x2 + y 2 ,
 0, se (x, y) = (0, 0)

admite derivadas parciais em (0, 0), mas não é contı́nua neste ponto.

Solução: De acordo com a definição de derivada parcial, temos:

∂f f (0 + ∆x, 0) − f (0, 0)
(0, 0) = lim
∂x ∆x→0 ∆x
∂f f (∆x, 0) − f (0, 0)
(0, 0) = lim =0
∂x ∆x→0 ∆x
e
∂f f (0, 0 + ∆y) − f (0, 0)
(0, 0) = lim
∂y ∆y→0 ∆y
∂f f (0, ∆y) − f (0, 0)
(0, 0) = lim =0
∂x ∆y→0 ∆x
∂f ∂f
Assim, f admite derivadas parciais em (0, 0) e valem (0, 0) = (0, 0) = 0.
∂x ∂y

Para mostrar que f não é contı́nua em (0, 0), verificaremos se lim f (x, y) existe.
(x,y)→(0,0)
Inicialmente determinaremos este limite ao longo da reta de equação y = x.

x·x x2 1
f (x, y) = f (x, x) = 2 2
= 2
= .
x +x 2x 2
1
Portanto, f (x, y) → 2
quando (x, y) → (0, 0) ao longo de y = x.
No entanto, se considerarmos este limite ao longo do eixo x, temos:
x·0 0
f (x, y) = f (x, 0) = = 2 = 0.
x2+0 2 x
Assim, f (x, y) → 0, quando (x, y) → (0, 0) ao longo do eixo y.
24 Meneses, L. R.; Noskoski, O. Cálculo II

Como f tem dois limites diferentes ao longo de duas retas distintas, concluı́mos que o
limite não existe. Assim, a função não é contı́nua em (0, 0).
A figura a seguir mostra o esboço gráfico da função f .

xy
Figura: Gráfico da função definida por f (x, y) =
x2 + y2

 3 2
 x − y , se (x, y) 6= (0, 0)
Exemplo 23 Seja f (x, y) = x2 + y 2 . Determine:

0, se (x, y) = (0, 0)
∂f ∂f
(a) (b)
∂x ∂y

Solução: (a) Nos pontos (x, y) 6= (0, 0) podemos aplicar a regra do quociente. Veja a
seguir.
∂ 3 ∂
∂f (x2 + y 2 ) (x − y 2 ) − (x3 − y 2 ) (x2 + y 2 )
(x, y) = ∂x ∂x
∂x (x2 + y 2 )2
(x2 + y 2 ) · 3x2 − (x3 − y 2 ) · 2x
=
(x2 + y 2 )2
3x4 + 3x2 y 2 − 2x4 + 2xy 2
=
(x2 + y 2 )2
x4 + 3x2 y 2 + 2xy 2
=
(x2 + y 2 )2

Em (0, 0), temos:

∂f f (0 + ∆x, 0) − f (0, 0)
(0, 0) = lim
∂x ∆x→0 ∆x
∆x3
2
−0
= lim ∆x
∆x→0 ∆x
∆x − 0
= lim =1
∆x→0 ∆x
25 Meneses, L. R.; Noskoski, O. Cálculo II

∂f
Assim, é função de R2 em R dada por
∂x
 4
x + 3x2 y 2 + 2xy 2
∂f  , se (x, y) 6= (0, 0)
(x, y) = (x2 + y 2 )2
∂x 
1, se (x, y) = (0, 0)

(b) Nos pontos (x, y) 6= (0, 0) podemos aplicar a regra do quociente. Veja a seguir.

∂ 3 ∂
(x2 + y 2 ) (x − y 2 ) − (x3 − y 2 ) (x2 + y 2 )
∂f ∂y ∂y
(x, y) = 2 2 2
∂y (x + y )
(x + y ) · (−2y) − (x3 − y 2 ) · 2y
2 2
=
(x2 + y 2 )2
−2x y − 2y 3 + 2x3 y + 2y 3
2
=
(x2 + y 2 )2
−2x2 y(1 + x)
=
(x2 + y 2 )2

Em (0, 0), temos:

∂f f (0, 0 + ∆y) − f (0, 0)


(0, 0) = lim
∂y ∆y→0 ∆y
−1 − 0
= lim
∆y→0 ∆y
−1 ∂f ∂f
Note que, lim não existe, ou seja, (0, 0) não existe. Assim, está definida em
∆y→0 ∆y ∂y ∂y
todo (x, y) 6= (0, 0) e é dada por:

∂f −2x2 y(1 + x)
(x, y) =
∂y (x2 + y 2 )2

x3 − y 2
Figura: Gráfico da função definida por f (x, y) =
x2 + y 2
26 Meneses, L. R.; Noskoski, O. Cálculo II

3.2 Interpretação Geométrica


Suponha que C1 é a intersecção da superfı́cie z = f (x, y) com o plano y = y0 e que C2
é a sua intersecção com o plano x = x0 . Assim, fx (x, y0 ) pode ser interpretada como a
taxa de variação de z em relação a x ao longo da curva C1 , e fy (x, y) pode ser interpre-
tada como a taxa de variação de z em relação a y ao longo da curva C2 . Em particular,
∂f
(x0 , y0 ) é a taxa de variação de z em relação a x ao longo da curva C1 no ponto (x0 , y0 ) e
∂x
∂f
(x0 , y0 ) é a taxa de variação de z em relação a y ao longo da curva C2 no ponto (x0 , y0 ).
∂y

Geometricamente, fx (x0 , y0 ) pode ser vista como o coeficiente angular da reta tangente à
curva C1 no ponto (x0 , y0 ); e fy (x0 , y0 ) pode ser vista como o coeficiente angular da reta
tangente à curva C2 no ponto (x0 , y0 ).

Exemplo 24 De acordo com a lei dos gases ideiais para um gás confinado, em P newtons
por metros quadrados for a pressão, V metros cúbicos for o volume e T graus for a
temperatura, teremos a fórmula
P V = kT (3.1)
onde k é uma constante de proporcionalidade. Suponha que o volume de um gás em certo
recipiente seja 100 m3 e que a temperatura seja 90◦ C e k = 8.

(a) Determine a taxa de variação de P por unidade de variação de T se V permanece


fixo em 100 m3 .

(b) Use o resultado do item (a) para aproximar a taxa de variação da pressão, se a
temperatura for aumentada para 92◦ C.

(c) Determine a taxa de variação de V por unidade de variação em P se T permanece


fixa em 90◦ C.
27 Meneses, L. R.; Noskoski, O. Cálculo II

Solução:
(a) Considerando a equação 3.1 e k = 8, temos:

8T
P =
V
Assim, a taxa de variação de P por unidade de variação de T se V permanece fixo em
100 m3 é obtida derivando esta equação com relação a T e considerando V constante e
igual a 100 m3 . Logo
∂P 8 8
= = = 0, 08
∂T V 100
∂P
(b) = 0, 08, significa que para cada unidade de temperatura aumentada a pressão
∂T
aumenta de 0,08. Assim, se a temperatura passar de 90◦ C para 92◦ C, ou seja, sofrer um
aumento de 2◦ C, o aumento aproximado em P será de 2 · 0, 08 = 0, 16 N/m2 .

8T
(c) Sendo V = , temos:
P
∂V 8T
=− 2
∂P P
8 · 90
Note que, para V = 100 m3 , T = 90◦ C e k = 8, obtemos P = = 7, 2 N/m2 . Logo,
100
a taxa de variação de V por unidade de variação em P quando T = 90◦ C e P = 7, 2, se
T permanecer fixa em 90◦ C é dada por
∂V 8 · 90 125
=− 2
=−
∂P 7, 2 9

3.3 Diferenciação Parcial Implı́cita


Definição 17 Uma função z = f (x, y) se diz definida implicitamente pela equação
g(x, y, z) = 0 se, para todo (x, y) pertencente ao domı́nio de f tivermos g(x, y, f (x, y)) =
0.

p
Considere por exemplo, a função f , definida por z = f (x, y) = 9 − x2 − y 2 , com
x2 + y 2 < 9, é dada implicitamente pela equação x2 + y 2 + z 2 = 9 pois, para todo
p
(x, y) no seu domı́nio, x2 + y 2 + ( 9 − x2 − y 2 )2 = 9.

Para efetuar a diferenciação implı́cita, consideramos z como uma função de x e y e dife-


renciamos ambos os lados em relação a x, mantendo y fixo, ou em relação a y, mantendo
x fixo.
28 Meneses, L. R.; Noskoski, O. Cálculo II

Exemplo 25 Sendo z = f (x, y) dada implicitamente por x2 + y 2 + z 2 = 9, com z > 0,


∂f ∂f
calcule e .
∂x ∂y
Solução:
∂ 2 ∂ ∂ 2 ∂
(x + y 2 + z 2 ) = (9) (x + y 2 + z 2 ) = (9)
∂x ∂x ∂y ∂y
∂z ∂z
2x + 0 + 2z =0 0 + 2y + 2z =0
∂x ∂y
∂z x ∂z y
=− =−
∂x z ∂y z

∂z ∂z
Exemplo 26 Calcule e usando a diferenciação implı́cita, sendo x2 + z sin xyz = 0.
∂x ∂y
Solução:
∂ 2 ∂
(x + z sin xyz) = (0)
 ∂x ∂x
∂z ∂z
2x + z · cos xyz · xy + zy + sin xyz · = 0
∂x ∂x
∂z
(xyz cos xyz + sin xyz) = −2x − yz 2 cos xyz
∂x
∂z −2x − yz 2 cos xyz
=
∂x xyz cos xyz + sin xyz

∂ 2 ∂
(x + z sin xyz) = (0)
∂y ∂y
 
∂z ∂z
0 + z · cos xyz · xy + zx + sin xyz · = 0
∂x ∂y
∂z
(xyz cos xyz + sin xyz) = −xz 2 cos xyz
∂y
∂z −xz 2 cos xyz
=
∂y xyz cos xyz + sin xyz

Exemplo 27 Se resistores elétricos de R1 , R2 e R3 ohms são conectados em paralelo para


formar um resistor de R ohms, o valor de R pode ser encontrado a partir da equação
1 1 1 1
= + + .
R R 1 R2 R3
∂R
Encontre o valor de quando R1 = 30, R2 = 45 e R3 = 90 ohms.
∂R2
∂R
Solução: Para determinarmos , tratamos R1 e R3 como constantes e diferenciamos
∂R2
ambos os lados da equação em relação a R2 . Veja abaixo.
29 Meneses, L. R.; Noskoski, O. Cálculo II

   
∂ 1 ∂ 1 1 1
= + +
∂R2 R ∂R2 R1 R2 R3
1 ∂R 1
− 2 = 0− 2 +0
R ∂R2 R2
2
 2
∂R R R
= 2 =
∂R2 R2 R2

Quando R1 = 30, R2 = 45 e R3 = 90,


1 1 1 1 3+2+1 6 1
= + + = = = .
R 30 45 90 90 90 15
Logo, R = 15 e  2
∂R 15 1
= =
∂R2 45 9

Exemplo 28 Ache a inclinação da reta tangente à curva de intersecção da superfı́cie



36x2 − 9y 2 + 4z 2 + 36 = 0 com o plano x = 1, no ponto (1, 12, −3). Faça um esboço do
gráfico. Interprete essa inclinação como a derivada parcial.

Solução: Sabemos que a inclinação da reta tangente à curva de intersecção da superfı́cie


de equação z = f (x, y) com um plano x = x0 , no ponto (x0 , y0 , f (x0 , y0 )) é dada pela
derivada parcial de f com relação a y, aplicada no ponto (x0 , y0 , f (x0 , y0 )). Como z
está definido implicitamente pela equação 36x2 − 9y 2 + 4z 2 + 36 = 0, devemos realizar a
∂z
derivação implı́cita para determinar . Considerando z função de x e de y, temos:
∂y
∂ ∂
(36x2 − 9y 2 + 4z 2 + 36) = (0)
∂y ∂y
∂z
−18y + 8z = 0
∂y
∂z 9y
=
∂y 4z
Assim, a inclinação da reta tangente à curva de intersecção da superfı́cie de equação

36x2 − 9y 2 + 4z 2 + 36 = 0 com um plano x = 1, no ponto (1, 12, −3) é dada por
√ √
∂z √ 9 12 3 12
(1, 12) = =−
∂y 4(−3) 4
30 Meneses, L. R.; Noskoski, O. Cálculo II

3.4 Derivadas Parciais de Segunda Ordem


Se f é uma função de duas variáveis, suas derivadas parciais fx e fy são funções de
duas variáveis, de modo que podemos considerar novamente suas derivadas parciais.
Isto origina quatro possı́veis derivadas parciais de segunda ordem de f , as quais são
definidas por
 
∂2 ∂ ∂ fx (x + ∆x, y) − fx (x, y)
f (x, y) = f (x, y) = lim
∂x2 ∂x ∂x ∆x→0 ∆x
 
∂2 ∂ ∂ fx (x, y + ∆y) − fx (x, y)
f (x, y) = f (x, y) = lim ,
∂y∂x ∂y ∂x ∆y→0 ∆y
 
∂2 ∂ ∂ fy (x + ∆x, y) − fy (x, y)
f (x, y) = f (x, y) = lim ,
∂x∂y ∂x ∂y ∆x→0 ∆x
 
∂2 ∂ ∂ fy (x, y + ∆y) − fy (x, y)
f (x, y) = f (x, y) = lim .
∂y 2 ∂y ∂y ∆y→0 ∆y

Resumindo, dizemos que a derivada segunda de f é a derivada da função derivada.


Também utilizamos a seguinte notação para as derivadas de segunda ordem:
∂2 ∂ 2f
f (x, y) = fxx (x, y) (x, y) = fxy (x, y)
∂x2 ∂y∂x

∂ 2f ∂ 2f
(x, y) = fyy (x, y) (x, y) = fyx (x, y)
∂y 2 ∂x∂y

Exemplo 29 Determine as derivadas parciais de segunda ordem de f , definida por


y
f (x, y) = (x2 + y 2 ) arctan .
x
Solução:
 
   y ′ 
∂ ∂f ∂  2 2 1 y 
fxx = = (x + y ) · · + arctan · 2x
∂x ∂x ∂x y2 x x
1+ 2
  x  
2
∂ 2 2 x x · 0 − y·1 y
= (x + y ) · 2 · + 2x arctan
∂x x + y2 x2 x
∂  y 
= −y + 2x arctan
∂x x
1  y ′  y
= 0 + 2x · · + arctan ·2
y2 x x
1+ 2
x 
2
x −y y
= 2x · 2 2
· 2
+ 2 arctan
x +y x x
−2xy y
= 2 2
+ 2 arctan
x +y x
31 Meneses, L. R.; Noskoski, O. Cálculo II
 
   y ′ 
∂ ∂f ∂  2 2 1 y 
fyy = = (x + y ) · · + arctan · 2y 
∂y ∂y ∂y y2 x x
1+ 2
 2
  x 
∂ 2 2 x 1 y
= (x + y ) · 2 · + 2y arctan
∂y x + y2 x x
∂  y 
= x + 2y arctan
∂y x
1 1  y
= 0 + 2y · · + arctan ·2
y2 x x
1+ 2
x
x2 1 y
= 2y · 2 · + 2 arctan
x + y2 x x
2xy y
= 2 2
+ 2 arctan
x +y x

 
∂ ∂f ∂  y
fxy = = −y + 2x arctan
∂y ∂x ∂y x
1 1  y 
= −1 + 2x · 2 · + arctan ·0
y x x
1+ 2
x
x2 1
= −1 + 2x · 2 2
·
x +y x
−x − y + 2x2
2 2
=
x2 + y 2
x2 − y 2
=
x2 + y 2

 
∂ ∂f ∂  y
fyx = = x + 2y arctan
∂x ∂y ∂x x
1 −y  y
= 1 + 2y · · + arctan ·0
y 2 x2 x
1+ 2
x
2
x −y
= 1 + 2y · 2 2
· 2
x +y x
2 2 2
x + y − 2y
=
x2 + y 2
x − y2
2
=
x2 + y 2
Note que no exemplo anterior fxy = fyx . O próximo teorema, do matemático fracês Alexis
Clairant, fornece condições sob as quais podemos afirmar que fxy = fyx .
32 Meneses, L. R.; Noskoski, O. Cálculo II

Teorema 3 Suponha que f seja definida em uma bola aberta D que contenha o ponto
(x0 , y0 ). Se as funções fxy e fyx forem ambas contı́nuas em D, então

fxy (x0 , y0 ) = fyx (x0 , y0 )

Em outras palavras, se as derivadas mistas de f forem contı́nuas no aberto D, então elas


são iguais, ou seja, não vai importar a ordem de derivação.

3.4.1 Aplicações
Equação de Laplace

A equação de Laplace tridimensional

∂ 2f ∂ 2f ∂ 2f
+ + =0
∂x2 ∂y 2 ∂z 2

é satisfeita pelas distribuições de temperatura no estado estacionário T = f (x, y, z) no


espaço, pelos potenciais gravitacionais e pelos potenciais eletrostáticos. A equação de
Laplace bidimensional
∂ 2f ∂ 2f
+ =0
∂x2 ∂y 2
∂ 2f
obtida eliminando-se o termo da equação anterior, descreve potenciais e distribuições
∂z 2
de temperatura no estado etacionário no plano.

Equação da Onda

Se ficarmos em uma praia e tirarmos uma fotografia das ondas, esta mostrará um padrão
regular de picos e depressões em dado instante. Veremos movimento vertical periódico no
espaço em relação à distância. Se ficarmos na água, poderemos sentir a subida e descida
da água com o passar das ondas. Veremos movimento periódico vertical no tempo. Em
fı́sica, essa bela simetria é expressa pela equação de onda unidimensional

∂ 2w 2
2∂ w
= c
∂t2 ∂x2
onde w é a altura da onda, x é a variável distância, t é a variável tempo e c é a velocidade
com a qual as ondas se propagam.

Em nosso exemplo, x é a distância ao longo da superfı́cie do mar, mas em outras aplicações


x pode ser a distância ao longo de uma corda vibrando, a distância no ar (ondas sonoras)
ou a distância no espaço (ondas luminosas). O número c varia de acordo com o meio e o
tipo de onda.
33 Meneses, L. R.; Noskoski, O. Cálculo II

3.5 Derivadas Parciais de Ordem Superior


Apesar de lidarmos na maioria das vezes com derivadas parciais de primeira e segunda
ordens, porque elas aparecem com mais freqı̈¿ 21 ência em aplicações, não existe limite
teórico para o número de vezes que podemos diferenciar uma função desde que as derivadas
envolvidas existam. Assim, obtemos derivadas parciais de terceira ordem que denotamos
por
∂ 3f
= fyyx
∂x∂y 2
∂ 4f
= fyyxx
∂x2 ∂y 2

e assim, por diante. Como acontece com derivadas de segunda ordem, a ordem de dife-
renciação é irrelevante desde que as derivadas na ordem em questão sejam contı́nuas.

3.6 Diferenciabilidade
Você deve lembrar do trabalho com funções de uma variável real que se f , definida por
y = f (x), for uma função derivável, então:

∆y
f ′ (x) = lim
∆x→0 ∆x

onde ∆y e ∆x são incrementos de x e y e

∆y = f (x + ∆x) − f (x).

∆y
Quando |∆x| for pequeno e ∆x 6= 0, difere de f ′ (x) por um número pequeno que
∆x
depende de |∆x| e será denotado por ǫ. Então,

∆y
ǫ= − f ′ (x) se ∆x 6= 0
∆x
onde ǫ é uma função de ∆x. Dessa forma, podemos reescrever a equação acima com:

∆y = f ′ (x)∆x + ǫ∆x.

Note que, ǫ → 0 quando ∆x → 0.


Quando isto ocorre para funções de uma variável real dizemos que a mesma é diferenciável
em x.

Para funções de duas ou mais variáveis, uma equação correspondente a esta é usada
para definir a diferenciabilidade da função.
34 Meneses, L. R.; Noskoski, O. Cálculo II

Definição 18 Se f for uma função de duas variáveis x e y, então o incremento de f


no ponto (x0 , y0 ), denotado por ∆f (x0 , y0 ), é dado por

∆f (x0 , y0 ) = f (x0 + ∆x, y0 + ∆y) − f (x0 , y0 )

Definição 19 Se f for uma função de duas variáveis x e y e o incremento de f em


(x0 , y0 ) puder ser escrito como

∆f (x0 , y0 ) = fx (x0 , y0 )∆x + fy (x0 , y0 )∆y + ǫ1 ∆x + ǫ2 ∆y,

onde ǫ1 e ǫ2 são funções de ∆x e ∆y, tais que ǫ1 → 0 e ǫ2 → 0 quando (∆x, ∆y) → (0, 0),
então dizemos que f é diferenciável em (x0 , y0 ).

Teorema 4 Se uma função f de duas variáveis for diferenciável em um ponto, ela será
contı́nua neste ponto.

Prova: Se f for diferenciável em (x0 , y0 ), temos:

f (x0 + ∆x, y0 + ∆y) − f (x0 , y0 ) = fx (x0 , y0 )∆x + fy (x0 , y0 )∆y + ǫ1 ∆x + ǫ2 ∆y

f (x0 + ∆x, y0 + ∆y) = f (x0 , y0 ) + fx (x0 , y0 )∆x + fy (x0 , y0 )∆y + ǫ1 ∆x + ǫ2 ∆y


Tomando o limite de ambos os membros da equação quando (∆x, ∆y) → (0, 0), obtemos

lim f (x0 + ∆x, y0 + ∆y) = f (x0 , y0 ).


(∆x,∆y)→(0,0)

Note que, considerando x = x0 + ∆x e y = y0 + ∆y, então (∆x, ∆y) → (0, 0) é equivalente


a (x, y) → (x0 , y0 ). Logo,
lim f (x, y) = f (x0 , y0 )
(x,y)→(x0 ,y0 )

o que prova a continuidade da função.

Observação 5 O Teorema 4 estabelece que para uma função de duas variáveis,


diferenciabilidade implica continuidade. No entanto, a existência de derivadas parciais
num ponto não implica diferenciabilidade naquele ponto. Verifiquemos isto no exemplo
abaixo.

Exemplo 30 No exemplo 22, mostramos que a função f , definida por


 xy
 , se (x, y) 6= (0, 0)
f (x, y) = x2 + y 2 ,
 0, se (x, y) = (0, 0)

admite derivadas parciais em (0, 0), no entanto, não é contı́nua neste ponto. Logo, con-
cluı́mos que f não é diferenciável em (0, 0).
35 Meneses, L. R.; Noskoski, O. Cálculo II

3.7 Regra da Cadeia


Considerando-se o Cálculo de funções de uma variável real, quando y = f (x) era uma
função diferenciável de x e x = g(t) era uma função diferenciável de t, y tornava-se uma
função diferenciável de t e a regra da cadeia dizia que
dy dy dx
= ·
dt dx dt
Para funções de duas ou mais variáveis, a regra da cadeia possui diversas formas que de-
pendem da quantidade de variáveis envolvidas. A seguir, são apresentadas algumas delas.

Teorema 5 (Derivada Total)


Se z = f (x, y) possui derivadas parciais contı́nuas fx e fy e se x = x(t) e y = y(t)
forem funções diferenciáveis de t, então a composta z = f (x(t), y(t)) será uma função
diferenciável de t e
df dx dy
= fx (x(t), y(t)) + fy (x(t), y(t))
dt dt dt
ou
df ∂f dx ∂f dy
= + ,
dt ∂x dt ∂y dt
df
onde chama-se Derivada Total de f .
dt

Prova: Devemos mostrar que, se x e y forem diferenciáveis em t = t0 , então f será dife-


renciável em t0 .

Consideremos ∆x, ∆y e ∆f os incrementos que resultam da variação de t0 a t0 + ∆t.


Como f é diferenciável, podemos escrever
∂f ∂f
∆f (x(t0 ), y(t0 )) = (x(t0 ), y(t0 ))∆x + (x(t0 ), y(t0 ))∆y + ǫ1 ∆x + ǫ2 ∆y (3.2)
∂x ∂y
onde ǫ1 → 0 e ǫ2 → 0 quando (∆x, ∆y) → (0, 0).

df
Para encontrarmos , dividimos ambos os membros da equação por ∆t e tomamos o
dt
limite quando ∆t → 0. Assim, dividindo 3.2 por ∆t e, em seguida, tomando o limite
quando ∆t → 0, no ponto P0 (x(t0 ), y(t0 )), obtemos:

         
∆f ∂f ∆x ∂f ∆y
lim = lim + lim +
∆t→0 ∆t P0 ∂x P0
∆t→0 ∆t ∂y P0
∆t→0 ∆t

  ∆x   ∆y
+ lim ǫ1 lim + lim ǫ2 lim
∆t→0 ∆t→0 ∆t ∆t→0 ∆t→0 ∆t
36 Meneses, L. R.; Noskoski, O. Cálculo II

Logo,          
df ∂f dx ∂f dy dx dy
= + +0· +0·
dt P0 ∂x P0 dt t0 ∂y P0 dt t0 dt dt
         
df ∂f dx ∂f dy
= + .
dt P0 ∂x P0 dt t0 ∂y P0 dt t0
Isto prova o teorema.

O diagrama a seguir mostra um esquema prático para montar a derivada total. Bem
acima, indicamos f , a função dada. De f , partem duas ramificações, chegando em x e
y, variáveis principais. Como x e y são, ainda, funções de t, o esquema termina com as
ramificações de x e de y “migrando” para t.
Assim, cada malha será um produto e a soma das duas malhas resulta na igualdade
estabelecida pelo teorema.

Exemplo 31 Determine a derivada total de f , utilizando a regra da cadeia, sendo


x+t 1
f (x, y) = , x = ln t e y = ln .
y+t t

Solução:
df ∂f dx ∂f dy
= +
dt ∂x dt ∂y dt
 
1
df 1 1

(y + t) · 0 − (x + t) · 1
 −
 2
= · + · t 
dt y+t t (y + t)2 1
  t
df 1 (x + t) 1
= − 2
· −
dt t(y + t) t(y + t) t
df y+t+x+t
=
dt t(y + t)2
df x + y + 2t
=
dt t(y + t)2
37 Meneses, L. R.; Noskoski, O. Cálculo II

Teorema 6 Se w = f (x, y, z) for diferenciável e x, y e z forem funções diferenciáveis


de t, então w será uma função derivável de t e
dw ∂w dx ∂w dy ∂w dz
= + +
dt ∂x dt ∂y dt ∂z dt

O diagrama a seguir mostra um esquema prático para determinar a derivada total de w.

dw
Exemplo 32 Determine a derivada total , utilizando a Regra da Cadeia, sendo w =
dt
p π
x2 + y 2 + z 2 , x = tan t, y = cos t e z = sin t, com 0 < t < .
2
Solução:

dw ∂w dx ∂w dy ∂w dz
= + +
dt ∂x dt ∂y dt ∂z dt
dw x y z
=p (sec2 t) + p (− sin t) + p (cos t)
dt x2 + y 2 + z 2 x2 + y 2 + z 2 x2 + y 2 + z 2

Exemplo 33 Num dado instante, o comprimento de um cateto de um triângulo retângulo


é 10 cm e ele está crescendo a uma taxa de 1 cm/min e o comprimento do outro cateto é
12 cm o qual está descrescendo a uma taxa de 2 cm/min. Determine a taxa de variação
da medida do ângulo agudo oposto ao lado de 12 cm de comprimento, num dado instante.
Solução: Considere o triângulo retângulo ABC, de catetos x e y que medem respecti-
vamente, 10 cm e 12 cm; e α o ângulo oposto ao cateto y, conforme ilustra a figura a
seguir.

C C

2 61
y 12

α α
A x B A 10 B
38 Meneses, L. R.; Noskoski, O. Cálculo II

Podemos escrever α em função de x e y, como


y
tan α = (3.3)
x
A taxa de variação da medida do ângulo agudo oposto ao lado de 12 cm num dado
instante é dada pela derivada parcial de α com relação ao tempo t. Logo, derivaremos
implicitamente a expressão 3.3 em relação a t.
∂ ∂ y
(tan α) =
∂t ∂t x
dα ∂f dx ∂f dy
sec2 α = +
dt ∂x dt ∂t dt
√ !2
2 61 dα y 1
= − 2 · 1 + · (−2)
10 dt x x
   
dα y 2 100
= − 2− ·
dt x x 4 · 61
   
dα y + 2x 100
= − ·
dt x2 4 · 61
   
dα −12 − 20 100
= ·
dt 102 4 · 61
dα 8
= −
dt 61
Assim, a taxa de variação da medida do ângulo agudo oposto ao lado de 12 cm num dado
8
instante é − rad/min.
61

Exemplo 34 A altura de um cilindro circular reto está decrescendo a uma taxa de


10 cm/min e o raio crescendo a uma taxa de 4 cm/min. Determine a taxa de variação do
volume no instante em que a altura é 50 cm e o raio é 16 cm.

Solução: Sabemos que V (r, h) = πr2 h, logo a taxa de variação do volume no instante em
∂V
que a altura é 50 cm e o raio é 16 cm é dada por (16, 50). Assim, temos:
∂t

dV ∂V dr ∂V dh
(r, h) = (r, h) + (r, h)
dt ∂r dt ∂h dt
dV
(r, h) = 2rπh · 4 + πr2 · (−10)
dt
dV
(16, 50) = 8π · 16 · 50 + π(16)2 (−10)
dt
dV
(16, 50) = 3840πcm3 /min
dt
39 Meneses, L. R.; Noskoski, O. Cálculo II

Exemplo 35 A voltagem V em um circuito que satisfaz a lei V = IR vai caindo lenta-


mente a uma taxa de 0, 01 volts/s a medida que a bateria descarrega. Ao mesmo tempo,
a resistência R vai aumentando a uma taxa de 0, 5 ohm/s, conforme o resistor esquenta.
Determine como a corrente está variando no instante em que R = 600 ohm e I = 0, 04 A.

Solução: A voltagem V é função de I e R, logo, podemos escrever:


dV ∂V dI ∂V dR
= +
dt ∂I dt ∂R dt
dV dI dR
= R +I
dt dt dt
dV dR
Como, = −0, 01 volts/s, = 0, 5 ohm/s, R = 600 ohm e I = 0, 04 A, obtemos:
dt dt
dI dI
−0, 01 = 600 · + 0, 04 · 0, 5 ⇒ = −0, 00005 A/s
dt dt

Teorema 7 (Regra da Cadeia)


Se f for uma função diferenciável de x e y definida por w = f (x, y), onde x = g(r, s) e
∂x ∂x ∂y ∂y
y = h(r, s) e , , , todas existirem, então w será uma função de r e s e
∂r ∂s ∂r ∂s
∂w ∂w ∂x ∂w ∂y
= +
∂r ∂x ∂r ∂y ∂r

∂w ∂w ∂x ∂w ∂y
= +
∂s ∂x ∂s ∂y ∂s
40 Meneses, L. R.; Noskoski, O. Cálculo II

∂w ∂w
Exemplo 36 Determine e , sendo w = arcsin(3x + y); x = r2 es e y = sin rs
∂s ∂r
Solução: A equação w = arcsin(3x + y) pode ser reescrita como sin w = 3x + y. Para
∂w ∂w ∂w ∂w
determinarmos e , precisamos inicialmente calcular e . Note que, w é dado
∂s ∂r ∂x ∂y
∂w ∂w
implicitamente pela equação sin w = 3x + y, logo para calcular e , utilizaremos a
∂x ∂y
derivação implı́cita.

∂ ∂ ∂ ∂
(sin w) = (3x + y) (sin w) = (3x + y)
∂x ∂x ∂y ∂y
∂w ∂w
cos w · =3 cos w · =1
∂x ∂y
∂w 3 ∂w 1
= =
∂x cos w ∂y cos w

∂w ∂w ∂x ∂w ∂y
= +
∂r ∂x ∂r ∂y ∂r
∂w 3 1
= · (2res ) + · (s cos rs)
∂r cos w cos w
∂w 6res + s cos(rs)
=
∂r cos w

∂w ∂w ∂x ∂w ∂y
= +
∂s ∂x ∂s ∂y ∂s
∂w 3 1
= · (r2 es ) + · (r cos rs)
∂s cos w cos w
∂w 3r2 es + r cos(rs)
=
∂s cos w
p
Considerando w a medida do arco cujo seno é 3x + y, obtemos cos w = 1 − (3x + y)2 .
Veja a figura a seguir.

1
3x + y
w
p
A 1 − (3x + y)2 B

Logo,
∂w 6res + s cos(rs) ∂w 3r2 es + r cos(rs)
=p = p
∂r 1 − (3x + y)2 ∂s 1 − (3x + y)2
41 Meneses, L. R.; Noskoski, O. Cálculo II

Teorema 8 (Regra da Cadeia Generalizada)


Suponha que f seja uma função diferenciável de n variáveis x1 , x2 , x3 , ..., xn e cada
uma dessas variáveis por sua vez seja uma função de m variáveis y1 , y2 , y3 , ..., ym .
∂xi
Suponha ainda qua cada uma das derivadas parciais (i = 1, 2, ..., n; j = 1, 2, ..., m)
∂yj
exista. Então, f é uma função de y1 , y2 , y3 , ..., ym , e
∂f ∂f ∂x1 ∂f ∂x2 ∂f ∂xn
= + + ... +
∂y1 ∂x1 ∂y1 ∂x2 ∂y1 ∂xn ∂y1

∂f ∂f ∂x1 ∂f ∂x2 ∂f ∂xn


= + + ... +
∂y2 ∂x1 ∂y2 ∂x2 ∂y2 ∂xn ∂y2
..
.
∂f ∂f ∂x1 ∂f ∂x2 ∂f ∂xn
= + + ... +
∂ym ∂x1 ∂ym ∂x2 ∂ym ∂xn ∂ym

∂w ∂w ∂w
Exemplo 37 Determine , e , sendo w = x2 + y 2 + z 2 , x = r sin φ cos θ,
∂r ∂θ ∂φ
y = r sin φ sin θ e z = r cos φ.

Solução:
∂w ∂w ∂x ∂w ∂y ∂w ∂z
= + +
∂r ∂x ∂r ∂y ∂r ∂z ∂r
∂w
= 2x(sin φ cos θ) + 2y(sin φ sin θ) + 2z cos φ
∂r
∂w ∂w ∂x ∂w ∂y ∂w ∂z
= + +
∂θ ∂x ∂θ ∂y ∂θ ∂z ∂θ
∂w
= −2xr sin φ sin θ + 2yr sin φ cos θ + 2z · 0
∂θ
∂w
= −2xr sin φ sin θ + 2yr sin φ cos θ
∂θ
42 Meneses, L. R.; Noskoski, O. Cálculo II

3.8 Derivadas Direcionais e o Vetor Gradiente


O mapa a seguir mostra os contornos da função temperatura T (x, y) às 19 h do dia 5
de agosto de 2009. As curvas de nı́vel ou isotérmas ligam localidades que apresentam
a mesma temperatura. A derivada parcial Tx em um ponto, como Bagé, dá a taxa de
variação da temperatura em relação a distância se viajarmos para leste; Ty é a taxa de
variação da temperatura se viajarmos para o norte. Mas o que acontece se desejarmos
conhecer a taxa de variação da temperatura em outra direção qualquer, por exemplo, para
nordeste, ou seja, se viajarmos de Bagé para Porto Alegre? Nesta seção introduziremos
um tipo de derivada, chamada derivada direcional, que nos permite determinar a taxa de
variação de uma função de duas ou mais variáveis em qualquer direção.

3.8.1 Derivadas Direcionais



− →

Suponha que a função f seja definida em uma região R no plano xy e que →

u = u1 i +u2 j
seja um vetor unitário arbitrário. Para determinar a taxa de variação de z no ponto (x0 , y0 )
na direção e sentido de →

u , devemos considerar a superfı́cie S com equação z = f (x, y) e
tomar z0 = f (x0 , y0 ). O ponto (x0 , y0 , z0 ) pertence a S. O plano vertical que passa por P
na direção de →

u intercepta S em uma curva C. A inclinação da reta tangente T a C em
P é a taxa de variação de z na direção e sentido de →

u.
43 Meneses, L. R.; Noskoski, O. Cálculo II

Se Q(x, y, z) é outro ponto sobre C e P ′ , Q′ são as projeções de P , Q sobre o plano xy,


−−→
então o vetor P ′ Q′ é paralelo a →

u , e portanto
−− →
P ′ Q′ = h→

u = (hu1 , hu2 )

para algum valor do escalar h. Dessa forma, x−x0 = hu1 e y−y0 = hu2 , logo x = x0 +hu1 ,
y = y0 + hu2 , e
∆z z − z0 f (x0 + hu1 , y0 + hu2 ) − f (x0 , y0 )
= =
h h h
Se tomarmos o limite quando h → 0, obteremos a taxa de variação de z em relação a
distância, na direção e sentido de →

u , que é chamada derivada direcional de f na direção
e sentido de →

u.

Isto nos motiva a definição que segue.

Definição 20 A derivada direcional de f em (x0 , y0 ) na direção e sentido do vetor



− →

unitário →

u = u i + u j é
1 2

f (x0 + hu1 , y0 + hu2 ) − f (x0 , y0 )


u f (x0 , y0 ) = lim
D−

h→0 h
se esse limite existir.



Observação 6 Comparando a definição 16 com 20, vemos que, se →

u = i = (1, 0),


então Di f = fx e se →

u = i = (0, 1), então Dj f = fy . Em outras palavras, as
derivadas parciais de f com relação a x e y são casos particulares da derivada direcional.

Outra notação para derivada direcional é a seguinte:


∂f
D−
u f (x0 , y0 ) = →
→ (x0 , y0 ).
∂−u

2
Exemplo
 38 Encontre
 a derivada de f (x, y) = x + xy em P (1, 2) na direção do vetor

− 1 →− 1 →−
u = √ i +√ j .
2 2
44 Meneses, L. R.; Noskoski, O. Cálculo II

Solução: Observe que →−


u é unitário. Assim, pela definição de derivada direcional, temos:
 
1 1
f x + h√ , y + h√ − f (x, y)
2 2
Du f (x, y) = lim
h→0 h
 2    
h h h
x+ √ + x+ √ · y+√ − (x2 + xy)
2 2 2
= lim
h→0 h
2xh h2 xh yh h2
x2 + √ + + xy + √ + √ + − x2 − xy
2 2 2 2 2
= lim
h→0 h
3x y
= lim √ + h + √
h→0 2 2
3x y
= √ +√
2 2

3·1 2 5 5 2
Du f (1, 2) = √ + √ = √ =
2 2 2 2

Teorema 9 Se f é uma função diferenciável em x e y, então f tem derivada direcional



− →

na direção e sentido de qualquer vetor unitário →

u = u1 i + u2 j e

Du f (x, y) = fx (x, y)u1 + fy (x, y)u2

Prova: Se definirmos uma função g de uma única variável real h por

g(h) = f (x0 + hu1 , y0 + hu2 )

então, pela definição de derivada ordinária (de uma variável), temos:


g(h) − g(0) f (x0 + hu1 , y0 + hu2 ) − f (x0 , y0 )
g ′ (0) = lim = lim ,
h→0 h h→0 h
ou seja,
g ′ (0) = Du f (x0 , y0 ). (3.4)

Por outro lado, podemos escrever g(h) = f (x, y), onde x = x0 + hu1 , y = y0 + hu2 , e
assim, pela regra da cadeia, obtemos
∂f dx ∂f dy
g ′ (h) = + = fx (x, y)u1 + fy (x, y)u2
∂x dh ∂y dh
Se tomarmos h = 0, então x = x0 , y = y0 e

g ′ (0) = fx (x0 , y0 )u1 + fy (x0 , y0 )u2 . (3.5)

Comparando 3.4 e 3.5, temos que

Du f (x0 , y0 ) = fx (x0 , y0 )u1 + fy (x0 , y0 )u2 (3.6)


45 Meneses, L. R.; Noskoski, O. Cálculo II

Se o vetor unitário →

u faz um ângulo θ com o eixo positivo, então podemos escrever

− →
− →

u = cos θ i + sin θ j e a fórmula 3.6 pode ser reescrita como

Du f (x, y) = fx (x, y) cos θ + fy (x, y) sin θ (3.7)

Isto prova o teorema.

y
u2

|~u|

θ
u1 x

1
Exemplo 39 Determine a derivada direcional da função f , definida por f (x, y) = ;
x−y
12 →
− 5→ −
na direção e sentido do vetor →

u =− i + j no ponto (−1, 2).
13 13
Solução: Note que, o vetor →
−u é unitário, assim inicialmente devemos determinar fx (x, y)
e fy (x, y).
 
∂ 1 (x − y) · 0 − 1 · 1 1
fx (x, y) = = 2
=−
∂x x − y (x − y) (x − y)2
 
∂ 1 (x − y) · 0 − 1 · (−1) 1
fy (x, y) = = =
∂y x − y (x − y)2 (x − y)2
Du f (x, y) = fx (x, y)u1 + fy (x, y)u2
   
1 12 1 5
Du f (x, y) = − 2
− + 2
(x − y) 13 (x − y) 13
17
Du f (x, y) =
13(x − y)2
17 17 17
Du f (−1, 2) = 2
= =
13(−1 − 2) 13 · 9 117
46 Meneses, L. R.; Noskoski, O. Cálculo II

3.9 Vetor Gradiente


Note que a derivada direcional pode ser escrita como o produto escalar de dois vetores.
Du f (x, y) = fx (x, y)u1 + fy (x, y)u2

− →
− →
− →

= (fx (x, y) i + fy (x, y) j ) · (u1 i + u2 j )

− →
− −
= (fx (x, y) i + fy (x, y) j ) · →
u

O primeiro vetor é muito importante e é chamado de gradiente da função f , denotado


por grad f ou ∇f (lê-se “nabla” de f , ou “del” de f ). ∇ é um operador diferencial veto-
rial. Sozinho, o ∇ não tem nenhum significado numérico; o significado surge quando ele
é aplicado a uma função.

Definição 21 Se f é uma função de duas variáveis x e y, o gradiente de f é a função


vetorial ∇f definida por
∂f →
− ∂f →

∇f (x, y) = i + j
∂x ∂y

Assim, podemos reescrever a expressão para derivada direcional como

Du f (x, y) = ∇f (x, y) · →

u

Para funções de três variáveis podemos definir derivadas direcionais de modo semelhante.

Definição 22 A derivada direcional de uma função f em (x0 , y0 , z0 ) na direção e sentido



− →
− →

do vetor unitário →

u = u i + u j + u k é
1 2 3

f (x0 + hu1 , y0 + hu2 , z0 + hu3 ) − f (x0 , y0 , z0 )


Du f (x0 , y0 , z0 ) = lim
h→0 h
se o limite existir.

Teorema 10 Se f é uma função diferenciável em x, y e z, então f tem derivada



− →
− →

direcional na direção e sentido de qualquer vetor unitário →

u = u1 i + u2 j + u3 k e

Du f (x, y, z) = fx (x, y, z)u1 + fy (x, y, z)u2 + fz (x, y, z)u3 (3.8)

A prova deste teorema é feita de forma análoga a do teorema 10.


47 Meneses, L. R.; Noskoski, O. Cálculo II

Para uma função f de três variáveis, o vetor gradiente, denotado por ∇f ou grad f , é
∂f →
− ∂f →
− ∂f →

∇f (x, y, z) = i + y + k
∂x ∂y ∂z
Logo, a equação 3.8 pode ser reescrita como

Du f (x, y, z) = ∇f (x, y, z) · →

u

Considere uma função f de duas ou três variáveis e todas as possı́veis derivadas direcionais
de f em um ponto dado. Isso nos dá a taxa de variação da função em todas as direções
possı́veis. Assim, podemos perguntar: Em qual dessas direções f varia mais rapidamente
e qual a máxima taxa de variação?

Se α for a medida em radianos do ângulo entre os dois vetores →



u e ∇f , então


u · ∇f (x, y) = |→

u ||∇f (x, y)| cos α

Assim, temos
Du f (x, y) = |→

u ||∇f (x, y)| cos α
Observe que, Du f é máxima quando cos α = 1, e isso ocorre quando θ = 0. Portanto, o
valor máximo de D f é |∇f | e ocorre quando →
u

u tem a mesma direção e mesmo sentido
que o vetor gradiente. Assim, o gradiente de uma função está na direção e sentido em que
a função tem a taxa máxima de variação. Geometricamente, isso significa que a superfı́cie
z = f (x, y) tem sua inclinação máxima em um ponto (x, y) na direção do gradiente e a
inclinação máxima é |∇f |. Analogamente, podemos dizer que o valor mı́nimo de Du f é
−|∇f |, e este valor ocorre quando θ = π, isto é, quando →

u está no sentido oposto a ∇f .
Geoemtricamente, isso significa que a superfı́cie z = f (x, y) tem sua inclinação mı́nima
em um ponto (x, y) no sentido oposto ao do gradiente, e a inclinação mı́nima é −|∇f |.

3.9.1 Propriedades Algébricas dos Gradientes


Considere k uma constante e os gradientes
∂f →
− ∂f →
− ∂f →
− ∂g →
− ∂g →
− ∂g →

∇f = i + j + k e ∇g = i + j + k,
∂x ∂y ∂z ∂x ∂y ∂z
o gradiente obedece as seguintes leis:

∇ (kf ) = k∇f
∇ (f + g) = ∇f + ∇f
∇ (f − g) = ∇f − ∇f
∇ (f g) = f ∇g − g∇f
 
f g∇f − f ∇g
∇ =
g g2
48 Meneses, L. R.; Noskoski, O. Cálculo II

Observação 7 Repare que pensando em funções de uma variável real t, temos que
d
∇ = . Logo, não é por acaso a semelhança das propriedades do gradiente com as
dt
regras usuais de derivação estudadas no Cálculo Diferencial.

Exemplo 40 Seja f , uma função definida por f (x, y) = x2 − 4y, determine:


(a) o gradiente de f em P (−2, 2);
π→− π→−
(b) a taxa de variação dos valores funcionais na direção de →

u = cos i + sin j no
3 3
ponto (−2, 2).

Solução:
(a)

∂f →− ∂f →

∇f (x, y) = i + j
∂x ∂y
∂ 2 →
− ∂ 2 →

= (x − 4y) i + (x − 4y) j
∂x ∂y

− →

= 2x i − 4 j

− →
− →
− →

∇f (−2, 2) = 2 · (−2) i − 4 j = −4 i − 4 j

(b)

Du f (x, y) = ∇f (x, y) · →

u
√ !

− →
− 1→− 3
= (2x i − 4 j ) · i +
2 2

= x−2 3

Du f (−2, 2) = −2 − 2 3

Exemplo 41 Esboce o mapa de contorno da função f , definida no exemplo 40, mostrando


as curvas de nı́vel em −8, −4, 0, 4, 8. Mostre também a representação de ∇f (−2, 2),
tendo seu ponto inicial em (−2, 2)

Solução: As curvas de nı́vel desta função tem a forma x2 − 4y = k. Assim, temos:


x2
k = −8 ⇒ x2 − 4y = −8 ⇒ y= +2
42
x
k = −4 ⇒ x2 − 4y = −4 ⇒ y= +1
4 2
x
k=0 ⇒ x2 − 4y = 0 ⇒ y=
4
x2
k=4 ⇒ x2 − 4y = 4 ⇒ y= −1
42
x
k=8 ⇒ x2 − 4y = 8 ⇒ y= −2
4
49 Meneses, L. R.; Noskoski, O. Cálculo II

Exemplo 42 O potencial elétrico V (x, y) volts em qualquer ponto do plano xy é V (x, y) =


e−2x cos 2y. A distância é medida em metros.
 π
(a) Determine a taxa de variação do pontencial no ponto 0, , na direção do vetor
4
π→− π→−
unitário cos i + sin j .
6 6

π
(b) Determine a direção, o sentido e o valor da taxa de variação máxima de V em 0, .
4
Solução:
(a)
  "√ #
∂ −2x →
− ∂ −2x →
− 3→− 1→−
Du V (x, y) = (e cos 2y) i + (e cos 2y) j · i + j
∂x ∂y 2 2
√ −2x
= − 3e cos 2y − e−2x sin 2y
 π √ π π
Du V 0, = − 3e−2·0 cos 2 · − e−2·0 sin 2 · = −1
4 4 4
 π
(b) A direção e o sentido da variação máxima de V em 0, são dados pelo vetor
 π 4
gradiente. Calculando-se ∇f 0, , temos:
4

 π π→− π→−
∇f 0, = −2e−2·0 cos 2 · i − 2e−2·0 sin 2 · j
4 4 4


= −2 j
 π →

Logo, a variação máxima de V se dá na direção e sentido do vetor ∇f 0, = −2 j e
 π  4

o valor da taxa de variação máxima de V é ∇f 0, = 2 volts/m
4
50 Meneses, L. R.; Noskoski, O. Cálculo II

3.10 Valores Extremos de Funções de Duas Variáveis


Uma aplicação importante da derivada de uma função de uma variável real consiste no
estudo dos valores extremos de uma função que nos leva a uma variedade de problemas
envolvendo máximos e mı́nimos. Nesta seção, veremos como usar as derivadas parciais
para localizar os pontos de máximo e mı́nimo de uma função de duas variáveis.

Definição 23 Seja f uma função de duas variáveis reais definida em uma região R
que contém o ponto (x0 , y0 ). Então, diz-se que f (x0 , y0 ) é um valor máximo relativo
(ou máximo local) de f se f (x0 , y0 ) ≥ f (x, y), para todo o ponto (x, y) do domı́nio de
f situado em um disco aberto centrado em (x0 , y0 ), e diz-se que f tem um máximo
absoluto em (x0 , y0 ) se f (x0 , y0 ) ≥ f (x, y) para todos os pontos (x, y) do domı́nio de f .

Definição 24 Seja f uma função de duas variáveis reais definida em uma região R
que contém o ponto (x0 , y0 ). Então, diz-se que f (x0 , y0 ) é um valor mı́nimo relativo
(ou mı́nimo local) de f se f (x0 , y0 ) ≤ f (x, y), para todo o ponto (x, y) do domı́nio de
f situado em um disco aberto centrado em (x0 , y0 ), e diz-se que f tem um mı́nimo
absoluto em (x0 , y0 ) se f (x0 , y0 ) ≤ f (x, y) para todos os pontos (x, y) do domı́nio de f .

Teorema 11 Se f tiver um valor de máximo ou mı́nimo relativo em um ponto (x0 , y0 )


do seu domı́nio e, se as derivadas parciais de primeira ordem de f existirem neste ponto,
então
fx (x0 , y0 ) = 0 e fy (x0 , y0 ) = 0
51 Meneses, L. R.; Noskoski, O. Cálculo II

Prova: Vamos provar que se f tiver um valor máximo relativo em (x0 , y0 ) e se fx (x0 , y0 )
existir, então fx (x0 , y0 ) = 0.

Pela definição de derivada parcial, temos

f (x0 + ∆x, y0 ) − f (x0 , y0 )


fx (x0 , y0 ) = lim
∆x→0 ∆x
Considerando que f tem um máximo relativo em (x0 , y0 ), temos:

f (x0 + ∆x, y0 ) − f (x0 , y0 ) ≤ 0

Se ∆x tender a zero pela direita, ∆x > 0, logo,

f (x0 + ∆x, y0 ) − f (x0 , y0 )


≤0
∆x
Logo, se fx (x0 , y0 ) existir, fx (x0 , y0 ) ≤ 0.

Analogamente, se ∆x tender a zero pela esquerda, ∆x < 0 e assim,

f (x0 + ∆x, y0 ) − f (x0 , y0 )


≥ 0.
∆x
Logo, se fx (x0 , y0 ) existir, fx (x0 , y0 ) ≥ 0.

Assim, concluı́mos que, se fx (x0 , y0 ) existir, ambas as desigualdades, fx (x0 , y0 ) ≤ 0 e


fx (x0 , y0 ) ≥ 0, devem ser satisfeitas. Consequentemente, segue que fx (x0 , y0 ) = 0.

A demonstração de que fy (x0 , y0 ) = 0 se fy (x0 , y0 ) existir e f tiver um valor máximo


retalivo em (x0 , y0 ) é análoga.

Definição 25 Um ponto (x0 , y0 ) é chamado de um ponto crı́tico da função f , se


fx (x0 , y0 ) = 0 e fy (x0 , y0 ) = 0, ou, se uma ou ambas derivadas parciais não existi-
rem em (x0 , y0 ).

Note que, da definição 25 e do teorema 11 segue que os extremos relativos ocorrem nos
pontos crı́ticos. No entanto, como no cálculo de funções de uma variável real, uma função
de duas variáveis reais não precisa ter um extremo relativo em cada ponto crı́tico.
52 Meneses, L. R.; Noskoski, O. Cálculo II

Exemplo 43 Determine os valores extremos da função f , definida por f (x, y) = y 2 − x2 .

Solução: Esta função, cujo gráfico é a superfı́cie S, que representa um parabolóide hi-
perbólico, tem um ponto crı́tico em (0, 0), pois

fx (x, y) = −2x e fy (x, y) = 2y


( (
fx (x, y) = 0 −2x = 0
⇒ ⇒ x=y=0
fy (x, y) = 0 2y = 0

Note que, a função f não admite ponto de máximo nem mı́nimo em (0, 0), pois consi-
derando C1 a curva que representa o traço da superfı́cie S com o plano vertical yz e C2
a curva que representa o traço da superfı́cie S com o plano xz, vemos que (0, 0) é um
ponto de mı́nimo de C1 e (0, 0) é um ponto de máximo de C2 . Neste caso, o ponto (0, 0)
é chamado ponto de sela da função f .

Teorema 12 Teste da Derivada Segunda


Seja f uma função de duas variáveis reais com derivadas parciais de segunda ordem
contı́nuas em uma bola aberta com centro em um ponto crı́tico (x0 , y0 ) e seja

D = D(x0 , y0 ) = fxx (x0 , y0 )fyy (x0 , y0 ) − [fxy (x0 , y0 )]2

(a) Se D > 0 e fxx (x0 , y0 ) > 0, então f tem um mı́nimo relativo em (x0 , y0 ).
(b) Se D > 0 e fxx (x0 , y0 ) < 0, então f tem um máximo relativo em (x0 , y0 ).
(c) Se D < 0, então f tem um ponto de sela em (x0 , y0 ).
(d) Se D = 0, então nenhuma conclusão pode ser tirada.

Observação 8 Para lembrar da fórmula de D é útil escrevê-la como um determinante



f f
xx xy
D=
fyx fyy

O determinante acima é chamado Hessiano da função f .

Exemplo 44 Determine os valores de máximo e mı́nimo relativo e os pontos de sela da


função f , definida por f (x, y) = x4 + y 4 − 4xy + 1, se estes existirem.

Solução: Inicialmente determinaremos os pontos crı́ticos de f . Assim, temos:

fx (x, y) = 4x3 − 4y e fy (x, y) = 4y 3 − 4x


( ( (
fx (x, y) = 0 4x3 − 4y = 0 x3 − y = 0
⇒ ⇒
fy (x, y) = 0 4y 3 − 4x = 0 y3 − x = 0
53 Meneses, L. R.; Noskoski, O. Cálculo II

Isolando-se x na equação (2) e substituindo-se na equação (1), obtemos:

(y 3 )3 − y = 0 ⇒ y9 − y = 0
⇒ y(y 8 − 1) = 0
⇒ y(y 4 − 1)(y 4 + 1) = 0
⇒ y(y 2 − 1)(y 2 + 1)(y 4 + 1) = 0

A equação acima admite como solução y = −1, y = 0 e y = 1, logo, os pontos crı́ticos de


f são: (−1, −1), (0, 0) e (1, 1).

Para o teste da derivada segunda, precisamos determinar as derivadas de segunda ordem



2
 fxx (x, y) = 12x

fyy (x, y) = 12y 2 ⇒ D = 144x2 y 2 − 16


fxy (x, y) = −4

Assim, obtemos a seguinte tabela:

Ponto Crı́tico D(x, y) = 144x2 y 2 − 16 fxx (x, y) = 12x2


(−1, −1) D = 144(−1)2 (−1)2 − 16 = 128 fxx = 12(−1)2 = 12
(0, 0) D = 144(0)2 (0)2 − 16 = −16 fxx = 12(0)2 = 0
(1, 1) D = 144(1)2 (1)2 − 16 = 128 fxx = 12(1)2 = 12

Note que, fxx > 0 e D > 0, nos pontos (−1, −1) e (1, 1), assim nestes pontos ocorrem
mı́nimos relativos e, como D < 0 no ponto (0, 0), concluı́mos que este é um ponto de sela
da função f . A figura a seguir, ilustra o gráfico da função f .

Figura: Gráfico da função definida por f (x, y) = x4 + y 4 − 4xy + 1


54 Meneses, L. R.; Noskoski, O. Cálculo II

Exemplo 45 Seja f , uma função definida por f (x, y) = x2 + y − ey . Localize todos os


pontos máximos e mı́nimos relativos e os pontos de sela de f , se estes existirem.

Solução: Inicialmente determinaremos os pontos crı́ticos. Assim, temos:

fx (x, y) = 2x e fy (x, y) = 1 − ey
( (
fx (x, y) = 0 2x = 0
⇒ ⇒ x=y=0
fy (x, y) = 0 1 − ey = 0

Logo, o único ponto crı́tico de f é (0, 0).

Agora, aplicaremos o teste da derivada segunda. Para isto, precisamos determinar as


derivadas parciais de segunda ordem de f .

 fxx = 2

fyy = −ey ⇒ D(x, y) = fxx (x, y) · fyy (x, y) − [fxy (x, y)]2 = −2ey


fxy = 0

Assim, fxx (0, 0) = 2 > 0 e D(x, y) = −2 < 0, logo (0, 0) é um ponto de sela da função f .
A figura a seguir, ilustra o gráfico da função f .

Figura: Gráfico da função definida por f (x, y) = x2 + y − ey

Exemplo 46 Seja f uma função definida por f (x, y) = ex sin y. Localize todos os pontos
máximos e mı́nimos relativos e pontos de sela, se estes existirem.

Solução: Determinaremos inicialmente os pontos crı́ticos de f .

fx (x, y) = ex sin y e fy (x, y) = ex cos y


( (
fx (x, y) = 0 ex sin y = 0

fy (x, y) = 0 ex cos y = 0
55 Meneses, L. R.; Noskoski, O. Cálculo II

Note que, o sistema acima não admite solução, pois ex > 0 e não existe em R um valor
para y tal que sin y = 0 e cos y = 0. Assim, f não possui pontos crı́ticos.
Sabemos pelo teorema 11 e pela definição 25, que os extremos relativos ocorrem nos pontos
crı́ticos. Logo, como a função f não possui pontos crı́ticos, não possui extremos relativos.

Figura: Gráfico da função definida por f (x, y) = ex cos x

3.10.1 Extremos Absolutos em Conjuntos Fechados e Limitados

Teorema 13 Teorema do Valor Extremo


Seja f uma função de duas variáveis x e y. Se f for contı́nua em um conjunto fechado
e limitado R, então f tem ambos máximo e mı́nimo absolutos em R.

Este teorema nos garante a existência do máximo e mı́nimo absoluto de f em um conjunto


R fechado e limitado. Esses extremos absolutos podem ocorrer ou na fronteira de R ou
no interior, então ele ocorre em um ponto crı́tico.

Para determinar um máximo ou mı́nimo absolutos de uma função contı́nua f em um


conjunto fechado e limitado R, adotaremos o seguinte procedimento:

(a) Determinar os pontos crı́ticos que estão situados no interior de R.


(b) Determinar todos os pontos de fronteira nos quais os extremos podem ocorrer.
(c) Calcular f (x, y) nos pontos obtidos nos passos (a) e (b). O maior desses valores é o
máximo absoluto e o menor valor é o mı́nimo absoluto.
56 Meneses, L. R.; Noskoski, O. Cálculo II

Exemplo 47 Determine os extremos absolutos da função f , definida por f (x, y) = xy −


x − 3y sobre a região triangular R com vértices (0, 0), (0, 4) e (5, 0).

Solução: A figura a seguir, representa a região R.

y
4

2
R
1

0 1 2 3 4 5 x

Como R é uma região fechada e limitada, pelo teorema do valor extremo, podemos afirmar
que f assume máximo e mı́nimo absolutos em R.
Inicialmente determinaremos os pontos crı́ticos de f . Assim, temos:

fx (x, y) = y − 1 e fy (x, y) = x − 3
( (
fx (x, y) = 0 y−1=0
⇒ ⇒ x=3 e y=1
fy (x, y) = 0 x−3=0

Logo, (3, 1) é o único ponto crı́tico de f e está no interior de R.

Aplicando-se o teste da derivada segunda, temos:



 fxx (x, y) = 0

fyy (x, y) = 0 ⇒ D(x, y) = fxx (x, y) · fyy (x, y) − [fxy (x, y)]2 = −1


fxy (x, y) = 1
Note que, neste caso, pelo teste da derivada segunda (3, 1) é um ponto de sela.

Agora, devemos determinar os pontos sobre a fronteira de R nos quais um valor extremo
pode ocorrer. A fronteira de R é constituı́da por três segmentos de reta. O segmento de
reta L1 , entre (0, 0) e (0, 4); o segmento de reta L2 , entre (0, 4) e (5, 0) e o segmento de
reta L3 , entre (5, 0) e (0, 0).

Em L1 , temos x = 0, logo

g(y) = f (0, y) = −3y, 0 ≤ y ≤ 4

Note que, trata-se de uma função decrescente de y, portanto seu máximo é f (0, 0) = 0 e
seu mı́nimo é f (0, 4) = −12.
57 Meneses, L. R.; Noskoski, O. Cálculo II

4
Em L2 , temos y = − x + 4, logo
5
     
4 4 4
h(x) = f x, − x + 4 = x · − x + 4 − x − 3 · − x + 4
5 5 5
4 27
h(x) = − x2 + x − 12, 0 ≤ x ≤ 5
5 5
8 27 27
Como h′ (x) = − x+ , a equação para h′ (x) admite x = como único ponto crı́tico de
5 5 8
27
h. Assim, os valores extremos de h ocorrem ou no ponto crı́tico x = ou nos extremos
8
x = 0 e x = 5. 
Os extremos
 correspondem aos pontos (0, 4) e (5, 0) de R, e o ponto crı́tico
27 13
corresponde a , .
8 10

Em L3 , temos y = 0, logo

j(x) = f (x, 0) = −x, 0 ≤ x ≤ 5

Trata-se de uma função decrescente de x, portanto seu máximo é f (0, 0) = 0 e seu mı́nimo
é f (5, 0) = −5.

A tabela a seguir apresenta os valores de f (x, y) nos pontos mencionados anteriormente.


 
27 13
(x, y) (0, 0) (0, 4) (5, 0) (1, 3) ,
8 10
231
f (x, y) 0 −12 −5 −7 −
80
Da tabela acima, concluı́mos que o valor máximo absoluto é f (0, 0) = 0 e o valor mı́nimo
absoluto é f (0, 4) = −12.

Exemplo 48 Determine os pontos de máximo e mı́nimo absolutos da função f , definida


2 2 y3
por f (x, y) = 3x + 2y + sobre a região Ω : {(x, y) ∈ R2 /x2 + y 2 ≤ 1}
9
Solução: A figura a seguir representa a região Ω.

y
1

−1 0 1 x

−1

Como Ω é um conjunto fechado e limitado, pelo teorema do valor extremo, podemos


afirmar que f a assume máximo e mı́nimo absolutos em Ω.
58 Meneses, L. R.; Noskoski, O. Cálculo II

Inicialmente determinaremos os pontos crı́ticos de f . Assim, temos:

y2
fx (x, y) = 6x e fy (x, y) = 4y +
3
( 
fx (x, y) = 0  6x = 0 x=0
⇒ y 2 ⇒
fy (x, y) = 0  4y + =0 y = 0 ou y = −12
3
Como −1 ≤ y ≤ 1, o único ponto crı́tico em Ω é (0, 0). Aplicando o teste da segunda
derivada, temos:

 f (x, y) = 6
 xx
 D(x, y) = fxx
(x, y) · fyy
(x, y) − [fxy (x, y)]
2
2
fyy (x, y) = 4 + y ⇒ 2

 3 D(x, y) = 6 · 4 + y

fxy (x, y) = 0 3

Como fxx (0, 0) = 6 > 0 e D(0, 0) = 24 > 0, concluı́mos que (0, 0) é um ponto de mı́nimo
relativo.

Agora, determinaremos os pontos sobre a fronteira de Ω nos quais um valor extremo pode
ocorrer. A fronteira de Ω consiste no conjunto dos pontos (x, y), tais que, x2 + y 2 = 1,
logo temos:
y3
f (x, y) = g(y) = 3(1 − y 2 ) + 2y 2 +
9
y3
g(y) = 3 − y 2 + , −1 ≤ y ≤ 1
9
y2
Note que, g ′ (y) = −2y + , assim a equação g ′ (y) = 0 admite y = 0 ou y = 6 como
3
soluções. No entanto, −1 ≤ y ≤ 1, logo, a única solução admissı́vel é y = 0.
Como x2 + y 2 = 1, para y = 0, temos x = −1 ou x = 1. Assim, (−1, 0) e (1, 0) são pontos
crı́ticos de g.
Também devemos considerar os extremos do intervalo de variação de y, ou seja, y = −1
e y = 1. Para y = −1 ou y = 1, temos x = 0.

A tabela a seguir, apresenta os valores de f (x, y) nos pontos mencionados.

(x, y) (0, 0) (0, −1) (0, 1) (−1, 0) (1, 0)


17 19
f (x, y) 0 3 3
9 9
17
Como f (0, 0) = 0 e f (0, −1) = , temos que (0, 0) é ponto de mı́nimo absoluto de f
9
em Ω. Os pontos (−1, 0) e (1, 0) são pontos de máximo absoluto de f em Ω com valor
máximo f (−1, 0) = f (1, 0) = 3.
59 Meneses, L. R.; Noskoski, O. Cálculo II

3.10.2 Problemas Aplicados


Exemplo 49 Uma caixa retangular tem um volume de 20 m3 . O material usado nos
lados custa R$ 1, 00 por metro quadrado, o material usado no fundo custa R$ 2, 00 por
metro quadrado e o usado na parte superior custa R$ 3, 00 por metro quadrado. Quais
as dimensões da caixa mais barata?

Solução: Sejam
x = comprimento da caixa (m);
y = largura da caixa (m);
z = altura da caixa (m);
S = área superficial da caixa (m2 );
V = volume da caixa (m3 );
C = custo da caixa (R$)

Assim, temos:
S(x, y, z) = 2xy + 2yz + 2xz
V (x, y, z) = x · y · z
C(x, y, z) = 5xy + 2yz + 2xz

Nosso objetivo é minimizar a função custo C, sujeita à restrição de volume

xyz = 20 (3.9)
20
De 3.9, obtemos z = , logo a função custo pode ser reescrita como
xy
40 40
C(x, y) = 5xy + +
x y
As dimensões x e y, na fórmula acima devem ser positivas. Logo, devemos determinar
o valor mı́nimo absoluto de C sobre a região para a qual x > 0 e y > 0. Como esta
região não é limitada, não temos nenhuma garantia de que um valor de mı́nimo absoluto
exista. No entanto, se houver um, ele ocorre num ponto crı́tico de C. Assim, inicialmente
determinaremos os pontos crı́ticos de C.
∂C 40 ∂C 40
(x, y) = 5y − 2 e (x, y) = 5x − 2
∂x x ∂y y
60 Meneses, L. R.; Noskoski, O. Cálculo II

  
∂C 40 8
 ∂x (x, y) = 0
  
  5y − 2 = 0
  y− 2 =0

x x
⇒ ⇒
 ∂C  40  8

 (x, y) = 0  5x − 2 = 0
  x− 2 =0

∂y y y
Isolando-se x na segunda equação do sistema acima e substituindo-se na primeira, obte-
mos:
8 y4
y− 2 = 0 ⇒ y−8· =0 ⇒ −y(y 3 − 8) = 0
8 64
y2

A equação acima admite como solução y = 0 ou y = 2. Como queremos y > 0, a única


solução admissı́vel é y = 2. Note que, para y = 2, obtemos x = 2.
Para verificar se realmente temos um mı́nimo relativo, aplicaremos o teste da derivada
segunda.
 2
 ∂ C 80

 (x, y) = 3  2 


 ∂x 2 x ∂ 2C ∂ 2C ∂ C

 ∂ 2C D(x, y) = (x, y) · (x, y) − (x, y)
80 ∂x2 ∂y 2 ∂x∂y
2
(x, y) = 3 ⇒

 ∂y y 80 80

 2 D(x, y) = 3 · 3 − 25

 ∂ C x y

 (x, y) = 5
∂x∂y

∂ 2C
Como (2, 2) = 10 > 0 e D(2, 2) = 75 > 0, temos que (2, 2) é um ponto de mı́nimo
∂x2
absoltuto. Para determinarmos a terceira dimensão da caixa, substituı́mos x = 2 e y = 2
40
na equação z = , obtendo z = 5. Logo, as dimensões da caixa mais barata são 2 m,
xy
2 m e 5 m.

Exemplo 50 De uma folha de metal com 27 cm de largura deseja-se obter uma calha
dobrando-se as bordas da folha de iguais quantidades de modo que as abas façam o
mesmo ângulo com a horizontal. Qual a largura das abas e qual o ângulo que devem fazer
a fim de ter uma capacidade máxima?

Solução: A figura a seguir mostra a seção transversal da calha.

Como nosso objetivo é determinar a largura das abas e o ângulo φ para que a capacidade
da calha seja máxima, devemos maximizar a área da seção transversal.
61 Meneses, L. R.; Noskoski, O. Cálculo II

Considerando-se S a área da seção transversal, temos:

(27 − 2x + 27 − 2x + 2x cos φ) · x sin φ


S(x, φ) =
2
S(x, φ) = 27x sin φ − 2x sin φ + x2 sin φ cos φ
2

27 π
Note que, neste caso, 0 < x < e0<φ< .
2 2
Determinaremos inicialmente os pontos crı́ticos da função S. Assim, temos:

∂S
(x, φ) = 27 sin φ − 4x sin φ + 2x sin φ cos φ
∂x
∂S
(x, φ) = 27x cos φ − 2x2 cos φ + x2 sin φ(− sin φ) + cos φ(x2 cos φ)
∂φ
∂S
(x, φ) = 27x cos φ − 2x2 cos φ − x2 + 2x2 cos2 φ
∂φ


∂S
 ∂x = 0

 (
27 sin φ − 4x sin φ + 2x sin φ cos φ = 0


 ∂S 27x cos φ − 2x2 cos φ − x2 + 2x2 cos2 φ = 0
 =0
∂φ
Resolvendo-se a primeira equação do sistema, obtemos:
27
cos φ = 2 − (3.10)
2x
Substituindo-se esta expressão na segunda equação do sistema, obtemos:
     2
27 2 27 2 2 27
27x 2 − − 2x 2 − − x + 2x 2 − = 0
2x 2x 2x
 
729 2 2 2 54 729
54x − − 4x + 27x − x + 2x 4 − + 2 = 0
2 x 4x
729 729
54x − − 4x2 + 27x − x2 + 8x2 − 108x + = 0
2 2
3x2 − 27x = 0
3x(x − 9) = 0 ⇒ x = 0 ou x = 9

Como queremos x > 0, a única solução admissı́vel é x = 9. Substituindo-se x = 9 na


1 π
equação 3.10, obtemos cos φ = , logo, φ = rad.
2 3
62 Meneses, L. R.; Noskoski, O. Cálculo II

π
Para verificar se (x, φ) = 9, é realmente um ponto de máximo da função S, aplicare-
3
mos o teste da derivada segunda.

∂ 2S
= −4 sin φ + 2 sin φ cos φ
∂x2
∂ 2S
= −27x sin φ + 2x2 sin φ + 4x2 cos φ(− sin φ)
∂φ2
∂ 2S
= −27x sin φ + 2x2 sin φ − 4x2 cos φ sin φ
∂φ2
∂ 2S
(x, φ) = 27 cos φ − 4x cos φ + 2x sin φ(− sin φ) + cos φ(2x cos φ)
∂x∂φ
∂ 2S
(x, φ) = 27 cos φ − 4x cos φ − 2x sin2 φ + 2x cos2 φ
∂x∂φ
Logo,
 2   √
 ∂ S π 3 3

 9, = −

 ∂x2 3 2 ∂ 2S ∂ 2S

∂ 2S



 2   √ D(x, φ) = · −
∂ S π 243 3 ∂x 2 ∂φ 2 ∂x∂φ
9, =− ⇒  π  1358
 ∂φ2 3 2

 D 9, = >0




2
∂ S  π  27 3 4
 9, =−
∂x∂φ 3 2

∂ 2S  π   π  π
Como 9, < 0 e D 9, > 0, concluı́mos que 9, é um ponto de máximo
∂x2 3 3 3
absoluto. Assim, para que se tenha a área da seção transversal máxima, √
a largura da aba
π 459 3
deve ser 9 cm e o ângulo φ = rad. A saber, a área máxima é S = cm3 .
3 4

3.11 Multiplicadores de Lagrange


No exemplo 49 da seção anterior, resolvemos o problema de minimizar o custo C de uma
caixa retangular de volume 20 m3 feita de determinado material, ou seja, minimizamos a
função C(x, y, z) = 5xy + 2yz + 2xz, sujeita à restrição xyz = 20. Para resolver este tipo
de problema podemos resolver a equação de restrição para uma variável em termos das
outras e substituir o resultado em C. Assim, obteremos uma nova expressão para C em
função de x e y. Determinando os pontos crı́ticos de C e aplicando o teste da derivada
segunda, minimizamos a função custo. No entanto, em alguns problemas pode ser difı́cil
(ou impossı́vel) resolver a equação restrita para uma das variáveis em termos das outras.
Nestes casos, em que queremos determinar os extremos de uma função sujeita a uma
restrição, podemos utilizar um método poderoso chamado Método dos Multiplicadores de
Lagrange, o qual será discutido nesta seção.
63 Meneses, L. R.; Noskoski, O. Cálculo II

Primeiramente vamos considerar um caso simples: maximizar ou minimizar uma função


de duas variáveis, com uma condição do tipo g(x, y) = 0, que determina uma curva no
plano.
O procedimento para esse caso é obtido a partir do resultado dado no próximo teorema.

Teorema 14 (Multiplicadores de Lagrange - Duas Variáveis, Uma Restrição)


Sejam f e g funções de duas variáveis com derivadas parciais de primeira ordem
contı́nuas em algum conjunto aberto contendo a curva de restrição g(x, y) = 0, e admi-
tamos que ∇g 6= 0 em qualquer ponto da curva. Se f tiver um extremo relativo restrito,
então este extremo ocorre em um ponto (x0 , y0 ) da curva de restrição no qual os vetores
gradientes ∇f (x0 , y0 ) e ∇g(x0 , y0 ) são paralelos; isto é, há um número real λ tal que

∇f (x0 , y0 ) = λ∇g(x0 , y0 ),

onde o escalar λ é chamado Multiplicador de Lagrange.

Prova: Sejam f e g funções de duas variáveis com derivadas parciais de primeira or-
dem contı́nuas em algum conjunto aberto contendo a curva de restrição g(x, y) = 0.
Se ∇g(x0 , y0 ) = 0, esse vetor é paralelo a qualquer vetor, em particular será paralelo a
∇f (x0 , y0 ). Podemos então supor ∇g(x0 , y0 ) 6= 0. Isso significa que
 
∂g ∂g
(x0 , y0 ), (x0 , y0 ) 6= (0, 0).
∂x ∂y
∂g
Suponhamos que 6= 0. Considerando-se a expressão g(x, y) = 0, podemos escrever y
∂y
como uma função de x numa vizinhança de (x0 , y0 ). Assim, temos pela regra da Cadeia
que:
∂g dy ∂g
(x0 , y0 ) (x0 ) + (x0 , y0 ) = 0
∂y dx ∂x
Logo,
∂g
dy (x0 , y0 )
(x0 ) = − ∂x (3.11)
dx ∂g
(x0 , y0 )
∂y
Agora consideremos a função f (x, y(x)). Essa é uma função de uma variável real qua
admite máximo ou mı́nimo em x = x0 . Assim, sua derivada anula-se neste ponto.
Calculando-se esta derivada pela regra da cadeia, obtemos:
∂f ∂f dy
(x0 , y0 ) + (x0 , y0 ) (x0 ) = 0 (3.12)
∂x ∂y dx
Substituindo 3.11 em 3.12, temos
∂g
∂f ∂f − (x0 , y0 )
(x0 , y0 ) + (x0 , y0 ) ∂x = 0.
∂x ∂y ∂g
(x0 , y0 )
∂y
64 Meneses, L. R.; Noskoski, O. Cálculo II

∂g
Multiplicando ambos os membros da equação por (x0 , y0 ), uma vez que supomos este
∂y
termo diferente de zero, obtemos:
∂g ∂f ∂f ∂g
(x0 , y0 ) · (x0 , y0 ) − (x0 , y0 ) · (x0 , y0 ) = 0.
∂y ∂x ∂y ∂x

Note que, tal igualdade equivale a



∂f ∂f
(x , y ) (x0 , y0 )
∂x 0 0 ∂y
∂g ∂g = 0.
(x0 , y0 ) (x0 , y0 )

∂x ∂y
Logo, temos que as duas linhas representam vetores paralelos, ou seja, ∃λ ∈ R, tal que

∇f (x0 , y0 ) = λ∇g(x0 , y0 ).

Isto mostra que ∇f (x0 , y0 ) e ∇g(x0 , y0 ) são paralelos, provando assim o teorema.

Exemplo 51 Determine os valores extremos que a função f , definida por f (x, y) = xy,
assume na circunferência de equação x2 + y 2 = 10.

Solução: Queremos maximizar a função f , definida por f (x, y) = xy, sujeita a restrição
x2 + y 2 = 10.

A restrição pode ser interpretada como g(x, y) = 0, onde g(x, y) = x2 + y 2 − 10.


Pelo teorema 14 sabemos que

∂f ∂g



 (x, y) = λ (x, y)
 ∂x ∂x


 ∇f (x, y) = λ∇g(x, y)
⇒ ∂f ∂g
(x, y) = λ (x, y)
 g(x, y) = 0 

 ∂y ∂y


 2
x + y 2 − 10 = 0
65 Meneses, L. R.; Noskoski, O. Cálculo II

Logo, podemos escrever: 




 y = λ · 2x

x = λ · 2y



 x2 + y 2 − 10 = 0

Considerando as duas primeiras equações do sistema, temos:

y = λ · 2x ⇒ x = 2λ(2λx) ⇒ 4λ2 x − x = 0
1
Assim, temos que x = 0 ou λ = ± . Consideremos agora estes dois casos.
2

Caso 1: Se x = 0, então x = y = 0. Mas (0, 0) não está na circunferência. Conse-


quentemente, x 6= 0.

1
Caso 2: Se x 6= 0, então λ = ± e x = ±y. Fazendo essa substituição na equação
2
g(x, y) = 0, temos:

(±y)2 + y 2 − 10 = 0 ⇒ 2y 2 = 10 ⇒ y = ± 5.
√ √ √ √
Logo, x = − 5 ou x = 5 e, y = − 5 ou y = 5.

A função f assume seus valores extremos na circunferência em quatro pontos, a saber:


√ √ √ √ √ √ √ √
P1 (− 5, − 5), P2 (− 5, 5), P3 ( 5, − 5) e P4 ( 5, 5).
√ √ √ √ √ √ √ √
Como f (− 5, − 5) = f ( 5, 5) = 5 e f ( 5, − 5) = f (− 5, 5) = −5, temos que
√ √ √ √
(− 5, − 5) e f ( 5, 5) são pontos de máximo de f na circunferência e o valor máximo
√ √ √ √
é 5, bem como (− 5, 5) e f ( 5, − 5) são pontos de mı́nimo de f e o valor mı́nimo é −5.

Interpretação Geométrica

é facil explicar a base geométrica do método de Lagrange para as funções de duas variáveis.
Então, vamos começar tentando determinar os valores extremos de f (x, y) sujeita a res-
trição da forma g(x, y) = 0. Em outras palavras, queremos achar os valores extremos de
f quando o ponto (x, y) pertencer à curva de nı́vel g(x, y) = 0. Consideremos f a função
definida no exemplo anterior e g(x, y) = x2 + y 2 − 10. A figura a seguir mostra as curvas
de nı́vel da função f e a curva de equação x2 + y 2 − 10 = 0. Essas curvas de nı́vel tem
√ √ √ √
equação xy = k, com k = −7, −5, −2 2, − 2, 2, 2 2, 5, 7. Note que, maximizar ou
minimizar f sujeita a restrição g(x, y) = 0 é determinar qual o maior ou menor valor
de k tal que a curva de nı́vel f (x, y) = k intercepte g(x, y) = 0. Pela figura vemos que
isso acontece quando estas curvas se tocam, ou seja, quando essas curvas tem uma reta
tangente em comum. Logo, os vetores gradientes são paralelos.
66 Meneses, L. R.; Noskoski, O. Cálculo II

√ √ √ √
∆g(− 5, 5) ∆g( 5, 5)

√ √
∆f ( 5, 5)
c
√ √
∆f (− 5, 5)
√ √
0 ∆f ( 5, − 5)

√ √
∆f (− 5, − 5)

√ √ √ √
∆g(− 5, − 5) ∆g( 5, − 5)

A seguir são apresentadas outras versões do teorema 14, dos multiplicadores de Lagrange,
para situações diferentes.

Teorema 15 (Multiplicadores de Lagrange - Três Variáveis, Uma Restrição)


Sejam f e g funções de três variáveis com derivadas parciais de primeira ordem contı́nuas
em algum conjunto aberto contendo a superfı́cies de restrição g(x, y, z) = 0 e admitamos
que ∇g 6= 0 em qualquer ponto desta superfı́cie. Se f tiver um extremo relativo restrito,
então este extremo ocorre em um ponto (x0 , y0 , z0 ) da superfı́cie de restrição no qual os
vetores gradientes ∇f (x0 , y0 , z0 ) e ∇g(x0 , y0 , z0 ) são paralelos, isto é, existe um número
λ tal que
∇f (x0 , y0 , z0 ) = λ∇g(x0 , y0 , z0 )

onde λ é chamado “multiplicador de Lagrange”.


Exemplo 52 Determine o ponto do elipsóide de equação x2 + 2y 2 + 3z 2 = 1 cuja soma
das coordenadas seja máxima.

Solução: Queremos maximizar a função f , definida por f (x, y, z) = x + y + z, sujeita a


restrição x2 + 2y 2 + 3z 2 = 1. A restrição pode ser interpretada como g(x, y, z) = 0, onde
g(x, y, z) = x2 + 2y 2 + 3z 2 − 1.
Pelo teorema 15 sabemos que

∂f ∂g

 (x, y, z) = λ (x, y, z)



 ∂x ∂x
 

 ∂f ∂g
 ∇f (x, y, z) = λ∇g(x, y, z)  (x, y, z) = λ (x, y, z)
⇒ ∂y ∂y
 g(x, y, z) = 0 
 ∂f ∂g

 (x, y, z) = λ (x, y, z)



 ∂z ∂z

 2 2 2
x + 2y + 3z − 1 = 0
67 Meneses, L. R.; Noskoski, O. Cálculo II

Logo, podemos escrever 



 1 = λ · 2x




 1 = λ · 4y


 1 = λ · 6z



 2
x + 2y 2 + 3z 2 − 1 = 0
Como λ 6= 0, resolvendo-se as 3 primeiras equações do sistema, temos:
1 1 1
x= , y= , z=
2λ 4λ 6λ
Substituindo-se este resultado na última equação do sistema, obtemos:
r
1 1 1 11 11
2
+ 2+ 2
−1=0 ⇒ λ2 = ⇒ λ=±
4λ 8λ 12λ 24 24
Assim, os candidatos a extremos são os pontos:
r r r ! r r r !
1 11 1 11 1 11 1 11 1 11 1 11
P1 , , e P2 − ,− ,− .
2 24 4 24 6 24 2 24 4 24 6 24
r r
11 11 11 11
Como f (P1 ) = e f (P2 ) = − , concluı́mos que o ponto do elipsóide cuja
12 24 12 24 r
11 11
soma das coordenadas é máxima é P1 e a soma é .
12 24

Exemplo 53 Encontre o ponto sobre o plano x + 2y + 3z = 13 mais próximo do ponto


(1, 1, 1).

Solução: Queremos minimizar a função d, definida por


p
d(x, y, z) = (x − 1)2 + (y − 1)2 + (z − 1)2 ,

sujeita a restrição x + 2y + 3z = 13. A restrição pode ser interpretada como g(x, y, z) = 0,


onde g(x, y, z) = x + 2y + 3z − 13.
Note que, o ponto de mı́nimo da distância é o mesmo ponto de mı́nimo do quadrado da
distância. Assim, para evitar radicais minimizaremos a função f (x, y, z) = (x − 1)2 + (y −
1)2 + (z − 1)2 .
Pelo teorema 15, sabemos que

∂f ∂g

 (x, y, z) = λ (x, y, z)



 ∂x ∂x
 

 ∂f ∂g
 ∇f (x, y, z) = λ∇g(x, y, z)  (x, y, z) = λ (x, y, z)
⇒ ∂y ∂y
 g(x, y, z) = 0 
 ∂f ∂g

 (x, y, z) = λ (x, y, z)



 ∂z ∂z

 2 2 2
x + 2y + 3z − 1 = 0
68 Meneses, L. R.; Noskoski, O. Cálculo II

Logo, podemos escrever





 2(x − 1) = λ · 1

 2(y − 1) = λ · 2


 2(z − 1) = λ · 3

 x + 2y + 3z − 13 = 0

Resolvendo-se as três primeiras equações do sistema, temos:


λ+2 3λ + 2
x= , y = λ + 1, z=
2 2
Substituindo-se este resultado na última equação do sistema, obtemos:
 
λ+2 3λ + 2
+ 2(λ + 1) + 3 − 13 = 0 ⇒ λ=1
2 2

Assim,
 oponto do plano de equação x + 2y + 3z = 13 mais próximo do ponto (1, 1, 1) é
3 5
, 2, .
2 2
Capı́tulo 4

Integrais Múltiplas

4.1 Integrais Duplas - Conceitos Preliminares


Definição 26 Seja o retângulo R = {(x, y) ∈ R2 /a ≤ x ≤ b e c ≤ y ≤ d}, onde
a < b e c < d. Sejam P1 : a = x0 < x1 < ... < xn = b e P2 : c = y0 < y1 <
... < ym = d partições de [a, b] e [c, d], respectivamente. O conjunto de retângulos
R1 , R2 , R3 , ..., Rnm forma uma partição de R e denota-se por P = (R1 , R2 , ..., Rnm ).

Definição 27 A norma de uma partição P , denotada por |P | é o comprimento da


maior diagonal dos retângulos Ri , i = 1, 2, ..., nm.

Definição 28 Um subconjunto D do R2 é limitado se existir um retângulo R, com


D ⊂ R.

4.2 Problema Motivador


Considere uma função f definida em um subconjunto limitado D do R2 , contı́nua e limi-
tada com f (x, y) ≥ 0, ∀(x, y) ∈ D.
Qual é o volume do sólido S, formado pelos pontos abaixo do gráfico de f e acima de D,
ou seja, S = {(x, y, z) ∈ R3 /(x, y) ∈ D e 0 ≤ z ≤ f (x, y)}?
70 Meneses, L. R.; Noskoski, O. Cálculo II

O procedimento para responder a esta pergunta, é apresentado a seguir.

Considere um retângulo R = {(x, y) ∈ R2 /a ≤ x ≤ b e c ≤ y ≤ d} que contém


D. Fazendo-se uma partição de R, ou seja, subdividindo R em retângulos menores
R1 , R2 , ..., Rnm formados por retas paralelas aos eixos coordenados que passam por pontos
de uma partição de [a, b] e de uma partição de [c, d], um “pedaço” retangular pequeno de
comprimento ∆x e largura ∆y possui área ∆A = ∆x∆y.

Escolhendo-se qualquer ponto arbitrário em cada sub-retângulo; denotando-se o ponto do


k−ésimo sub-retângulo Rk por (xk , yk ) e convencionando que f (xk , yk ) = 0 se (xk , yk ) 6∈ D,
tomamos o produto f (xk , yk )∆Ak , que é o volume do paralelepı́pedo de base Rk e altura
f (xk , yk ) quando (xk , yk ) ∈ D.

nm
X nm
X
Assim, a soma f (xk , yk )∆Ak é uma aproximação do volume de S. As somas f (xk , yk )∆Ak
k=1 k=1
estão definidas e são chamadas somas de Riemann de f , relativas a partição P .

Note que, essa aproximação em geral melhora sempre que diminuı́mos |P |. Logo, é natu-
ral definir o volume de S como sendo um limite dessas somas, quando |P | → 0.
71 Meneses, L. R.; Noskoski, O. Cálculo II

Dizemos que o número real L é o limite dessas somas para |P | → 0


nm
X
L = lim f (xk , yk )∆Ak , L ∈ R
|P |→0
k=1
nm

X

se dado ǫ > 0 arbitrário, existe δ > 0 tal que L − f (xk , yk )∆Ak < ǫ para qualquer

k=1
P com |P | < δ e qualquer escolha dos pontos (xk , yk ) ∈ Rk , k = 1, 2, 3, ..., nm.

Tal número L, quando existe é único e denomina-se, segundo Riemann, integral dupla
de f sobre D e indica-se por
Z Z nm
X
f (x, y)dA = lim f (xk , yk )∆Ak
D |P |→0
k=1

Z Z
Observação 9 No caso de f ≥ 0 em D, a integral dupla f (x, y)dA é, quando
D
existe, o “volume” do sólido S, mecionado anteriormente.

4.3 Funções Integráveis


Teorema 16 Seja D ∈ R2 um subconjunto limitado e com área, e seja f uma função
contı́nua em um retângulo que contém D. Então, f é integrável em D.

4.4 Propriedades da Integral


Teorema 17 Seja D ∈ R2 um subconjunto limitado, e f e g funções integráveis em D,
então

(a) f + g e kf são integráveis em D e vale


Z Z Z Z Z Z
(f + g)(x, y)dxdy = f (x, y)dxdy + g(x, y)dxdy
Z ZD D
Z Z D

(k · f )(x, y)dxdy = k · f (x, y)dxdy.


D D

Z Z
(b) f (x, y) ≥ 0 em D ⇒ f (x, y)dxdy ≥ 0.
D
Z Z Z Z
(c) f (x, y) ≤ g(x, y) em D ⇒ f (x, y)dxdy ≤ g(x, y)dxdy.
D D
72 Meneses, L. R.; Noskoski, O. Cálculo II

4.5 Integrais Iteradas - Teorema de Fubini


O cálculo de integrais duplas a partir de sua definição seria um processo trabalhoso e
quase impossı́vel, na prática, na maioria dos casos. Esse problema é resolvido usando-se
o teorema de Fubini.

A idéia do teorema é a seguinte: se f : R = [a, b] × [c, d] → R é uma função contı́nua e


positiva e se fx : [c, d] → R é definida por fx (y) = f (x, y), então a área, A(x), da seção
plana abaixo do gráfico de f e acima do plano xOy, com x fixado, será
Z d Z d
A(x) = fx (y)dy = f (x, y)dy
c c

Pelo princı́pio de Cavallieri, sabemos que o volume do sólido S = {(x, y, z) ∈ R3 /(x, y) ∈


D e 0 ≤ z ≤ f (x, y)} é igual a
Z b Z b Z d 
A(x)dx = f (x, y)dy dx
a a c

e,
Z pelas
Z discussões do parágrafo anterior, esse volume também deveria ser igual a
f (x, y)dxdy. Logo, terı́amos
R
Z Z Z b Z d 
f (x, y)dxdy = f (x, y)dy dx,
R a c

que nos diz que para calcular a integral dupla primeiramente calculamos a integral simples
de f em relação a y (mantendo x fixo) de c até d e depois integramos a função resultante,
Z d
A = A(x) = f (x, y)dy em relação a x, de a até b.
c

O mesmo argumento, com as variáveis trocadas nos permite concluir que:


Z Z Z d Z b
f (x, y)dxdy = B(y)dy, sendo B(y) = f (x, y)dx.
R c a
Z b Z d  Z d Z b 
As integrais f (x, y)dy dx e f (x, y)dx dy são chmadas integrais itera-
a c c a
das de f em R.
73 Meneses, L. R.; Noskoski, O. Cálculo II

Para simplificação eliminamos os parênteses na notação das integrais, escrevendo:


Z bZ d Z dZ b
f (x, y)dydx e f (x, y)dxdy.
a c c a

Z Z
2 +y
Exemplo 54 Calcule xex dxdy, sendo R = [0, 1] × [−1, 1].
R

Solução: A figura a seguir ilustra a região de integração.

y
1

−1 0 1 x
−1

Logo,
Z Z Z 1 Z 1
x2 +y 2 +y
xe dxdy = xex dxdy
R −1 0
Z 1  1
1 x2 +y
= e dy
−1 2 0
Z
1 1  1+y 
= e − ey dy
2 −1
1  1+y  1
= e − ey −1
2
1 2 
= e − e1 − (e0 − e−1 )
2
1 2 
= e − e1 + e−1 − 1
2
Exemplo 55 Calcule o volume do sólido S, delimitado acima pela superfı́cie de equação
z = x2 + y 2 , pelos planos coordenados e pelos planos x = 1, y = 1.
74 Meneses, L. R.; Noskoski, O. Cálculo II

Solução: A figura a seguir ilustra a região R de integração.

y
1

0 1 x

Z Z Z 1 Z 1
2 2
V = (x + y )dA = (x2 + y 2 )dxdy
R 0 0
Z 1 3 1
x 2
= + y x dy
0 3
Z 1  0
1
= + y 2 dy
0 3
 1
1 y3
= y+
3 3 0
2
= u.v.
3

4.6 Integrais duplas sobre regiões não retangulares


limitadas
Corolário 1 Sejam c(x) e d(x) duas funções contı́nuas em [a, b] e tais que, para todo
x em [a, b], c(x) ≤ d(x). Seja D o conjunto de todos (x, y) tais que a ≤ x ≤ b e
c(x) ≤ y ≤ d(x). Nestas condições, se f (x, y) for contı́nua em D, então
Z Z Z Z !
b d(x)
f (x, y)dxdy = f (x, y)dy dx.
D a c(x)
75 Meneses, L. R.; Noskoski, O. Cálculo II

Z Z
Exemplo 56 Calcule (x − y)dxdy, onde D é o semicı́rculo x2 + y 2 ≤ 1, x ≥ 0.
D

Solução: A figura a seguir apresenta a região de integração D.

y √
1 y= 1 − x2

0 1 x


−1 y = − 1 − x2

Note que, D pode ser descrita como


√ √
D = {(x, y) ∈ R2 /0 ≤ x ≤ 1 e − 1 − x2 ≤ y ≤ 1 − x2 }.

Assim, temos: √
Z Z Z 1 Z 1−x2
(x − y)dxdy = √
(x − y)dydx
D 0 − 1−x2
Z 1  2
√1−x2
y
= xy − √
dx
0 2 − 1−x2
Z√ 1
= 2x 1 − x2 dx
0 1
2p 2 3
= − (1 − x ) dx
3 0
2
=
3
Exemplo 57 Calcule o volume sólido S limitado pelos planos coordenados e pelas su-
perfı́cies de equações: x2 + z 2 = 1 e x + y = 1.

Solução: A figura a seguir ilustra o sólido S.


76 Meneses, L. R.; Noskoski, O. Cálculo II

Z Z √ Z 1 Z 1−x √
V = 1− x2 dA = 1 − x2 dydx
R 0 0
Z 1 h√ i1−x
= ( 1 − x2 )y dx
0
Z0 1 √
= ( 1 − x2 )(1 − x)dx
0
Z 1√ Z 1 √
= 2
1 − x dx − ( 1 − x2 )xdx
|0 {z } |0 {z }
I1 I2

Para resolver I1 , faremos uma substituição trigonométrica do tipo x = sin θ. Observe a


figura a seguir.

1
x

θ

1− x2

Assim, temos:
(
x = sin θ π
⇒ Para x = 0, temos θ = 0 e para x = 1, temos θ =
dx = cos θdθ 2

Z Z π
1 √ 2 p
I1 = 1− x2 dx = 1 − sin2 θ cos θdθ
0 0
Z π
2
= cos2 θdθ
0
Z π
1 2
= [cos 2θ + 1] dθ
0 2
 π
1 1 2 π
= sin 2θ + θ =
4 2 0 4

Resolvendo-se I2 , obtemos
Z  1
1 √ 1p 2 1
I2 = ( 1 − x2 )xdx = − (1 − x2 )3 · =−
0 2 3 0 3

Como V = I1 − I2 , temos:  
π 1
V = + u.v.
4 3
77 Meneses, L. R.; Noskoski, O. Cálculo II

Corolário 2 Sejam a(y) e b(y) duas funções contı́nuas em [c, d] e tais que, para todo
y em [c, d], a(y) ≤ b(y). Seja D o conjunto de todos (x, y) tais que c ≤ y ≤ d e
a(y) ≤ x ≤ b(y). Nestas condições, se f (x, y) for contı́nua em D, então
Z Z Z Z !
d b(y)
f (x, y)dxdy = f (x, y)dx dy.
D c a(y)

Z Z
Exemplo 58 Calcule (x − y)dxdy, onde D é o semicı́rculo x2 + y 2 ≤ 1, x ≥ 0.
D

Solução: A figura a seguir apresenta a região de integração D.

y
1
p
x= 1 − y2

0 1 x

−1

Note que, D pode ser descrita como


p
D = {(x, y) ∈ R2 /0 ≤ x ≤ 1 − y 2 e − 1 ≤ y ≤ 1}.

Assim, temos:

Z Z Z 1 Z √1−y2
(x − y)dxdy = (x − y)dxdy
D −1 0
Z 1  √ 2
1−y
x2
= − yx dy
−1 2 0
Z 1 
1 − y2 p
2
= − y 1 − y dy
−1 2
 1
1 y3 1 p
= y− + 2
(1 − y 2 )3
2 6 2 −1
78 Meneses, L. R.; Noskoski, O. Cálculo II

 
1 1 1 1 2
= − − − + =
2 6 2 6 3
Z Z √
1 2−x2
Exemplo 59 Inverta a ordem de integração na integral f (x, y)dydx, onde f
0 x
é contı́nua em R2 .

Solução: De acordo com os limites de integração, vemos que a região de integração é dada
por

R : {(x, y) ∈ R2 /0 ≤ x ≤ 1 e x ≤ y ≤ 2 − x2 },

a qual é representada pela figura a seguir.

y
√ √
2 y= 2 − x2
y=x

√ √
− 2 0 2 x


− 2

Neste caso, para inverter a ordem de integração deveremos dividir a região R em duas
sub-regiões, como ilustra a figura a seguir.

y
√ √
2 y= 2 − x2
y=x
R2

R1
√ √
− 2 0 2 x


− 2

Assim, temos:
R1 = {(x, y) ∈ R2 /0 ≤ y ≤ 1 e 0 ≤ x ≤ y}

e
√ p
R2 = {(x, y) ∈ R2 /1 ≤ y ≤ 2 e 0≤x≤ 1 − y2}
79 Meneses, L. R.; Noskoski, O. Cálculo II

Logo, podemos escrever:


Z Z √ Z Z Z Z
1 2−x2
f (x, y)dydx = f (x, y)dxdy + f (x, y)dxdy
0 x R1 R2

Z 1 Z y Z √
2 Z √1−y2
= f (x, y)dxdy + f (x, y)dxdy
0 0 1 0

4.7 Integrais Duplas em Coordenadas Polares


Em muitas situações para facilitar a descrição de regiões de integração faz necessária uma
mudança de variáveis. Apresentaremos nesta seção a mudança de variáveis para coorde-
nadas polares.

Definição 29 Uma região polar simples num sistema de coordenadas polares é uma
região compreendida entre dois raios, θ = α e θ = β, e duas curvas polares contı́nuas,
r = r1 (θ) e r = r2 (θ), onde as equações dos raios e das curvas polares satisfazem as
seguintes condições:
(a) α ≤ β (b) β − α ≤ 2π (c) 0 ≤ r1 (θ) ≤ r2 (θ)

As coordenadas polares (r, θ) de um ponto estão relacionadas com as coordenadas retan-


gulares pelas equações:
ρ2 = x 2 + y 2 x = ρ cos θ y = ρ sin θ

y
y P (ρ, θ) = (x, y)

θ
x x

Assim, para convertemos de coordenadas retangulares para coordenadas polares em uma


integral dupla, escrevemos x = ρ cos θ e y = ρ sin θ, usamos os limites de integração
apropriados para ρ e θ, e substituimos dA por ρdρdθ1 .

1
Podemos pensar nos retângulos polares “infinitesimais” como retângulos convencionais com dimensões
ρdθ e dρ e portanto com área dA = ρdρdθ.
80 Meneses, L. R.; Noskoski, O. Cálculo II

Z Z
2 +y 2
Exemplo 60 Calcular ex dxdy, sendo R : {(x, y) ∈ R2 /1 ≤ x2 +y 2 ≤ 16 e −x ≤
R
y ≤ x}.

Solução: A figura a seguir ilustra a região de integração R.

y
y=x

1 4 x

y = −x

Fazendo-se a mudança de variáveis para coordenadas polares, temos:


x = ρ cos θ
π π
y = ρ sin θ, com 1 ≤ ρ ≤ 4 e − ≤θ≤ .
4 4
Z Z Z π Z
4
4
x2 +y 2 2
e dxdy = eρ ρdρdθ
R − π4 1

Z π  4
4 1 ρ2
= e dρ
− π4 2 1
Z π
1 16 4
= (e − e1 )dρ
−π 2
 4  π4
1 16 1 π
= (e − e )ρ = (e16 − e1 ).
2 −π 4
4

Z Z p
Exemplo 61 Calcule x2 + y 2 dxdy, sendo R, o semicı́rculo R : {(x, y) ∈ R2 /(x −
R
1)2 + y 2 ≤ 1, y ≥ 0}.

Solução: A figura a seguir ilustra a região de integração R.

2 x

Fazendo-se a mudança de variáveis para coordenadas polares, temos:


81 Meneses, L. R.; Noskoski, O. Cálculo II

(
x = ρ cos θ
⇒ (x − 1)2 + y 2 = 1
y = ρ sin θ
x2 − 2x + 1 + y 2 = 1
x2 + y 2 − 2x = 0
ρ2 − 2ρ cos θ = 0
ρ = 2 cos θ, para ρ 6= 0

Assim, a região R pode ser escrita em coordenadas polares por R : {(θ, ρ) ∈ R2 /0 ≤ θ ≤


π
e 0 ≤ ρ ≤ 2 cos θ}.
2
Z Z p Z π Z
2
2 cos θ
x2 + y 2 dxdy = ρ2 dρdθ
R 0 0
Z π  2 cos θ
2 ρ3
= dθ
0 3 0
Z π
8 2
= cos3 θdθ
3 0
Z π
8 2
= cos θ(1 − sin2 θ)dθ
3 0
 π
8 sin3 θ 2
= sin θ −
3 3 0
 
8 1 16
= 1− =
3 3 9

Exemplo 62 Ache o volume do sólido no primeiro octante limitado pelo cone de equação
p
z = x2 + y 2 e pelo cilindro de equação x2 + y 2 = 3y.

Solução: A figura a seguir ilustra o cone e o cilindro.

O sólido está acima do semicı́rculo


 Rcuja fronteira tem equação x2 + y 2 = 3y ou, após
2
2 3 9 p
completar os quadrados, x + y − = , e abaixo do cone de equação z = x2 + y 2 .
2 4
82 Meneses, L. R.; Noskoski, O. Cálculo II

A figura a seguir apresenta a região de integração.

3 x

Fazendo-se a mudança de variáveis para coordendas polares, temos

x2 + y 2 = 3y ⇒ ρ2 = 3ρ sin θ ⇒ ρ = 3 sin θ

Assim, o disco R é dado por


n π o
R = (r, θ) / 0 ≤ θ ≤ e 0 ≤ r ≤ 3 sin θ .
2

Z Z p Z π Z
2
3 sin θ
V = x2 + y 2 dA = ρ · ρdρdθ
R 0 0
Z π  3 sin θ
2 ρ3
= dθ
0 3 0
Z π
2
= 9 sin3 θdθ
0
Z π
2
= 9 sin θ(1 − cos2 θ)dθ
0
 π
cos3 θ 2
= 9 − cos θ +
3 0
 
1
= −9 −1 + = 6 u.v.
3

Exemplo 63 Determine o volume de uma esfera de raio a, utilizando coordenadas pola-


res.

Solução: Consideremos o caso mais simples, ou seja, uma esfera de raio a e centro na
origem, a qual é expressa em coordenadas cartesianas como x2 + y 2 + z 2 = a2 .
83 Meneses, L. R.; Noskoski, O. Cálculo II

p
Note que, a equação que descreve o hemisfério superior é dada por z = a2 − x 2 − y 2 ,
de modo que o volume da esfera é
Z Z p
V =2 a2 − x2 − y 2 dA,
R

onde R é a região circular apresentada a seguir.


y
a

−a a x

−a

Fazendo-se a mudança de variáveis para coordenadas polares, obtemos


Z Z p Z 2π Z a p
V = 2 a2 − x2 − y 2 dA = a2 − ρ2 ρdρdθ
R 0 0
Z 2π  a
2 2 2 23
= − (a − ρ ) dθ
0 3 0
Z 2π
2 3
= a dθ
0 3
 2π
2 3
= aθ
3 0
4 3
= πa
3
84 Meneses, L. R.; Noskoski, O. Cálculo II

4.8 Área de Superfı́cie


Seja S a superfı́cie com equação z = f (x, y), onde f tem derivadas parciais contı́nuas.
Nesta seção calcularemos a área da superfı́cie S, cujo gráfico é uma função de duas
variáveis.

Condidere f (x, y) ≥ 0 e o domı́nio D de f uma região retangular.

Inicialmente faremos uma partição de D de modo que todos os sub-retângulos tenham a


mesma área ∆A = ∆x∆y, ou seja, a norma da partição é constante; e consideraremos o
ponto (xi , yj ) como o vértice do retângulo Rij que está mais próximo da origem.

Seja Pij (xi , yj , f (xi , yj )) o ponto de S que possui como projeção no plano xy o ponto
(xi , yj ).

Note que, o plano tangente a S em Pij é uma aproximação de S perto de Pij . As-
sim, a área ∆Tij da parte deste plano tangente que está acima de Rij é uma aproximação
X n Xm
da área ∆Sij da parte de S que está acima de Rij . Desse modo, a soma ∆Tij é
i=1 j=1
uma aproximação da área total de S, a qual melhora sempre que aumentamos o número
de retângulos. Logo, é natural definir o volume de S como sendo um limite destas somas,
quando m, n → ∞, se este limite existir.

Assim, podemos escrever


n X
X m
A(S) = lim ∆Tij (4.1)
m,n→∞
i=1 j=1


Considere → −a e b os vetores que começam em Pij e correspondem aos lados do parale-


logramo com área ∆Tij . Assim, ∆Tij = |→ −a × b |. Como fx (xi , yj ) e fy (xi , yj ) são as


inclinações das retas tangentes a S em P , com direções →
ij

a e b , temos:
85 Meneses, L. R.; Noskoski, O. Cálculo II


− →
− →

a = ∆x i + fx (xi , yj )∆x k

− →
− →

b = ∆y j + fy (xi , yj )∆y k
e →
− →
− →


i j
k

− →

a × b = ∆x 0 fx (xi , yj )∆x

0 ∆y fy (xi , yj )∆y

− →
− →

= ∆x∆y k − fx (xi , yj )∆x∆y i − fy (xi , yj )∆y∆x j

− →

= [−fx (xi , yj ) i − fy (xi , yj ) j + 1]∆x∆y
Logo, q
∆Tij = [fx (xi , yj )]2 + [fy (xi , yj )]2 + 1 ∆A
Assim, de ??, temos:
n X
X m q
A(S) = lim [fx (xi , yj )]2 + [fy (xi , yj )]2 + 1 ∆A
m,n→∞
i=1 j=1

Teorema 18 A área da superfı́cie com equação z = f (x, y), (x, y) ∈ D, onde fx e fy são
contı́nuas, é Z Z q
A(S) = fx2 + fy2 + 1 ∆A (4.2)
D

Exemplo 64 Determine a área da parte do parabolóide z = x2 + y 2 que está abaixo do


plano z = 9.
Solução: Note que, o traço do parabolóide no plano é a circunferência de equação x2 +y 2 =
9, logo a superfı́cie dada está acima da região circular de centro na origem e raio 3. Logo,
Z Z q Z Z p
A= 2 2
fx + fy + 1 ∆A = 1 + (2x)2 + (2y)2 dA
D D
Z Z p
= 1 + 4x2 + 4y 2 dA
D

O cálculo desta integral torna-se mais simples fazendo-se a mudança de variáveis para
coordenadas polares. Assim, temos:
Z Z p Z 2π Z 3 p
1 + 4(x2 + y 2 )dA = 1 + 4ρ2 ρdρdθ
D 0 0
Z 2π  3
1 2 2 23
= · (1 + 4ρ ) dθ
0 8 3 0
Z 2π h √ i
1
= 37 37 − 1 dθ
12 0
1 h √ i2π
= (37 37 − 1)θ
12 0
π √
= (37 37 − 1) u.a.
6
86 Meneses, L. R.; Noskoski, O. Cálculo II

4.9 Integrais Triplas - Conceitos Preliminares


Do mesmo modo como definimos integrais de funções de uma única variável e duplas para
funções de duas variáveis, definiremos integrais triplas para funções de três variáveis. Tra-
taremos inicialmente o caso mais simples, para o qual f é definida em um paralelepı́pedo:

B = {(x, y, z) ∈ R2 /a ≤ x ≤ b, c ≤ y ≤ d e e ≤ z ≤ f },

onde a < b, c < d e e < f .

z
B

Definição 30 Seja o paralelepı́pedo B = {(x, y, z) ∈ R2 /a ≤ x ≤ b, c ≤ y ≤ d e e ≤


z ≤ f }, onde a < b, c < d e e < f . Sejam ∆1 : a = x0 < x1 < x2 < ... < xm = b,
∆2 : c = y0 < y1 < y2 < ... < yn = d e ∆3 : e = z0 < z1 < z2 < ... < zr = f
partições de [a, b], [c, d] e [e, f ], respectivamente. O conjunto finito de paralelepı́pedos
B1 , B2 , ..., Bmnr formados pelos planos paralelos aos planos coordenados que passam
pelos pontos de partições de [a, b], [c, d] e [e, f ] é chamado partição de B e denota-se
por ∆ = (B1 , B2 , ..., Bmnr ).

z
B

Fazendo-se uma partição de B, observa-se que cada elemento Bijk da partição apresenta
volume ∆V = ∆x∆y∆z.
87 Meneses, L. R.; Noskoski, O. Cálculo II

Bijk

∆z

∆x
∆y
z
B

Assim, a soma tripla de Riemann


m X
X n X
r
f (x∗ijk , yijk
∗ ∗
, zijk )∆V,
i=1 j=1 k=1

onde o ponto de amostragem (x∗ijk , yijk


∗ ∗
, zijk ) está em Bijk .

Definição 31 A integral tripla de f na região B é dada por


Z Z Z m X
X n X
r
f (x, y, z)dV = lim f (x∗ijk , yijk
∗ ∗
, zijk )∆V,
B m,n,r→∞
i=1 j=1 k=1

se esse limite existir.


Observe que a integral tripla sempre existe se f for contı́nua. Na definição acima escolhe-
mos o ponto de amostragem como qualquer ponto de cada elemento da partição, mas, se
escolhermos o ponto (xi , yj , zk ), obteremos uma expressão com aparência mais amistosa
para a integral tripla:
Z Z Z m X
X n X
r
f (x, y, z)dV = lim f (xi , yj , zk )∆V.
B m,n,r→∞
i=1 j=1 k=1

Definição 32 Se f é contı́nua em uma região B = [a, b] × [c, d] × [e, f ], então


Z Z Z Z f Z d Z b
f (x, y, z)dV = f (x, y, z)dxdydz.
B e c a
88 Meneses, L. R.; Noskoski, O. Cálculo II

4.10 Funções Integráveis


Teorema 19 Seja B ∩R3 um subconjunto limitado e com volume, e f = f (x, y, z) uma
função limitada em B. Se f é contı́nua, exceto num conjunto de volume zero, então f
é integrável em B.

Considere a região descrita por desiguladades do tipo a ≤ x ≤ b, g1 (x) ≤ y ≤ g2 (x) e


u1 (x, y) ≤ z ≤ u2 (x, y), conforme a figura a seguir.

z z = u2 (x, y)

a z = u1 (x, y)

b
y
y = g1 (x)
x y = g2 (x)

Nessa região, é possı́vel reduzir a integral tripla a uma integral iterada por meio da equação
Z Z Z Z bZ g2 (x) Z u2 (x,y)
f (x, y, z)dV = f (x, y, z)dzdydx.
B a g1 (x) u1 (x,y)

Para regiões descritas por desegualdades do tipo h1 (y) ≤ x ≤ h2 (x), c ≤ y ≤ d e


u1 (x, y) ≤ z ≤ u2 (x, y), podemos escrever a integral tripla como:
Z Z Z Z d Z h2 (y) Z u2 (x,y)
f (x, y, z)dV = f (x, y, z)dzdxdy.
B c h1 (y) u1 (x,y)

z
z = u2 (x, y)

z = u1 (x, y)

c
d
y
x = h1 (y)
x x = h2 (y)
89 Meneses, L. R.; Noskoski, O. Cálculo II

Com as devidas adaptações temos também:


Z Z Z Z Z "Z u2 (y,z)
#
f (x, y, z)dV = f (x, y, z)dx dydz.
B B u1 (y,z)

y
x = u1 (y, z)
x x = u2 (y, z)

Z Z Z Z Z "Z u2 (x,z)
#
f (x, y, z)dV = f (x, y, z)dy dxdz.
B B u1 (x,z)

x = u2 (y, z)
R

x = u1 (y, z)
y
x

Observação 10 Note que, o volume de um sólido B, limitado em R3 , é dado por


Z Z Z
V (B) = 1dxdydz,
B

quando esta integral existe. Caso contrário dizemos que B não tem volume.
Z Z √ Z
3 9−z 2 x
Exemplo 65 Calcule a integral iterada xydydxdz.
0 0 0

Solução:
Z Z √ Z Z Z √  
3 9−z 2 x 3 9−z 2
y2
xydydxdz = x· dxdz
0 0 0 0 0 2
90 Meneses, L. R.; Noskoski, O. Cálculo II

Z Z √
3 9−z 2
x3
= dxdz
0 0 2
Z 3 4
√9−z2
1 x
= · dz
0 2 4 0
Z 3
1
= (9 − z 2 )2 dz
0 8
Z
1 3
= (81 − 18z 2 + z 4 )dz
8 0
 3
1 3 z5 81
= 81z − 6z + =
8 5 0 5

Exemplo 66 Determine o volume do sólido limitado pela superfı́cie x2 + z 2 = 16, pelo


plano x + y = 4 e pelos planos coordenados.

Solução: A figura a seguir ilustra a região em questão.

Observe que o domı́nio de integração é dado pelo conjunto D = {(x, y, z) ∈ R3 /0 ≤ x ≤



4, 0 ≤ y ≤ 4 − x e 0 ≤ z ≤ 16 − x2 }. Assim, temos:
Z Z Z √
4 4−x 16−x2
V = dzdydx
0 0 0
Z 4 Z 4−x √
2
= [z]0 16−x dydx
Z0 4 Z0 4−x √
= 16 − x2 dydx
0 0
Z 4 h √ i4−x
= y 16 − x 2 dx
0
Z0 4 h i √
= (4 − x) 16 − x2 dx
0
Z 4√ Z 4√
= 4 2
16 − x dx + 16 − x2 (−x)dx
| 0 {z } |0 {z }
I1 I2
91 Meneses, L. R.; Noskoski, O. Cálculo II

Para resolver I1 , faremos uma substituição trigonométrica do tipo x = 4 sen θ. Observe a


figura a seguir:

4
x

θ

16 − x2

Assim, temos:

x = 4 sen θ θ
⇒ Para x = 0, temos θ = 0 e para x = 4, temos θ = .
dx = 4 cos θdθ 2

Z Z π
4 √ 2 √
I1 = 4 16 − x2 dx = 4 16 − 16 sen 2 θ(4 − cos θ)dθ
0 0
Z π
2
= 64 cos2 θdθ
0
Z π
1 2
= 64 [cos 2θ + 1] dθ
0 2
  π2
1
= 32 sen 2θ + θ = 16π
2 0

Calculando I2 , temos:

Z 4 √
I2 = 16 − x2 (−x)dx
0
Z
1 4√
= 16 − x2 (−2x)dx
2 0
1 2 hp i4 64
= · (16 − x2 )3 = −
2 3 0 3

Assim, temos:  
64
V = I1 + I2 = 16π − u.v.
3
92 Meneses, L. R.; Noskoski, O. Cálculo II

4.11 Mudança de Variáveis em Integrais Múltiplas


4.11.1 Integral Dupla
No cálculo de integrais de funções de uma variável real um dos métodos utilizados foi o
de substituição de variáveis, que é baseado na fórmula
Z b Z d
f (x)dx = f (g(u))g ′ (u)du,
a c

onde g é uma função com derivada contı́nua em um intervalo I que contém c e d, e de


modo que g(c) = a e g(d) = b. Além disso, supomos f contı́nua na imagem de g.

Também vimos que a mudança de variáveis é útil em integrais duplas. De fato, já utili-
zamos a mudança para coordenadas polares. As novas variáveis r e θ são relacionadas as
variáveis em coordenadas retangulares x e y, como

x = r cos θ y = sen θ

e a fórmula de mudança de varı́aveis fica


Z Z Z Z
f (x, y)dA = f (r cos θ, sen θ)rdrdθ,
B S

onde S é a região no plano rθ que corresponde à região B no plano xy.

De modo geral, podemos considerar uma mudança de variáveis em R2 dada pela trans-
formação T do plano uv no plano xy

T (u, v) = (x, y),

onde x e y estão relacionados com u e v pelas equações

x = x(u, v) y = y(u, v).

Note que, T é uma função cujo domı́nio e imagem são ambos subconjuntos de R2 . Se
T (u1 , v1 ) = (x1 , y1 ), então o ponto (x1 , y1 ) é denominado imagem do ponto (u1 , v1 ).

Vamos supor T de classe C 1 , ou seja, x = x(u, v) e y = y(u, v) admitem derivadas parciais


de primeira ordem contı́nuas, e T injetora, o que significa que não existem dois pontos
com a mesma imagem. A figura a seguir ilustra o efeito de uma transformação T em uma
região S do plano uv em uma região R no plano xy denominada imagem de S.
93 Meneses, L. R.; Noskoski, O. Cálculo II

v y

S R
(xi , yi )
(ui , vi )

u x

Exemplo 67 Considere a transformação T , definida por T (u, v) = (x, y) = (u2 −v 2 , 2uv).


Determine a imagem do quadrado S = {(u, v)/0 ≤ u ≤ 1, 0 ≤ v ≤ 1}.

Solução: Note que, a transformação T leva a fronteira de S na fronteira da imagem. Assim,


inicialmente consideraremos as fronteiras S1 : v = 0, 0 ≤ u ≤ 1, S2 : u = 1, 0 ≤ v ≤ 1,
S3 : v = 1, 0 ≤ u ≤ 1 e S4 : u = 0, 0 ≤ v ≤ 1.

S3
1

S4 S S2

0 S1 1 u

(a) Considerando a lei da transformação, temos para a região S1 :



 x = u2
→ 0 ≤ x ≤ 1.
y = 0

Assim, a imagem de S1 pela transformação T é o segmento de reta com extremos


nos pontos (0, 0) a (1, 0) no plano xy.

(b) Para a região S2 , temos:



x = 1 − v 2 y2
→ x=1− , 0 ≤ x ≤ 1,
y = 2v 4

que é um ramo da parábola.

(c) Para a região S3 , temos:



 x = u2 − 1 y2
→ x= − 1, −1 ≤ x ≤ 0,
y = 2u 4

que também é um ramo da parábola.


94 Meneses, L. R.; Noskoski, O. Cálculo II

(d) Considerando a lei da transformação, temos para a região S4 :



x = −v 2
→ −1 ≤ x ≤ 0.
y = 0

Assim, a imagem de S1 pela transformação T é o segmento de reta com extremos


nos pontos (−1, 0) a (0, 0) no plano xy.
A imagem da região S é a região R cujo esboço é apresentado na figura a seguir.

Agora, iremos verificar como utilizar a mudança de variáveis na integração dupla. Inicia-
remos com um pequeno retângulo S no plano uv cujo vétice inferior esquerdo é o ponto
(u0 , v0 ) e cujas dimensões são ∆u e ∆v.

A imagem de S é a região R do plano xy, de modo que T (u0 , v0 ) = (x0 , y0 ). O vetor


− →
− →

r (u, v) = x(u, v) i + y(u, v) j

é o vetor posição da imagem do ponto (u, v). A equação do lado inferior de S é v = v0 ,


cuja curva imagem é dada pela função vetorial →

r (u, v0 ). O vetor tangente em (x0 , y0 ) a
essa curva imagem é

− ∂x →
− ∂y →

ru = i + j.
∂u ∂u
95 Meneses, L. R.; Noskoski, O. Cálculo II

Da mesma forma, o vetor tangente em (x0 , y0 ) à curva imagem do lado esquerdo de S é


− ∂x →
− ∂y →

rv = i + j.
∂v ∂v
Podemos aproximar a região R pelo paralelogramo determinado pelos vetores secantes

− →

a =→

r (u0 + ∆u, v0 ) − →

r (u0 , v0 ) b =→

r (u0 , v0 + ∆v) − →

r (u0 , v0 ),

conforme ilustra a figura a seguir.

r(u0 , v + ∆v)

b
r(u0 , v0 )
a
r(u + ∆u, v0 )

Note que, podemos escrever




r (u0 + ∆u, v0 ) − →

r (u0 , v0 )


ru = lim
∆u→0 ∆u
e, assim,


r (u0 + ∆u, v0 ) − →

r (u0 , v0 ) ≈ ∆u→

ru .
Da mesma forma, podemos escrever:


r (u0 , v0 + ∆v) − →

r (u0 , v0 )


rv = lim
∆v→0 ∆u
e, assim,


r (u0 , v0 + ∆v) − →

r (u0 , v0 ) ≈ ∆v →

rv .
Observe que podemos aproximar R por um paralelogramo determinado pelos vetores
∆u→

ru e ∆v →

rv . Portanto, podemos aproximar a área de R pela área desse paralelogramo,
a qual é dada por
|∆u→

ru × ∆v →

rv | = | →

ru × →

rv |∆u∆v.
Calculando o produto vetorial, obtemos:

− →
→→
− −
i
j k
∂x ∂y ∂x ∂x
∂x ∂y

∂u ∂u
→−
∂u ∂v



0 =
k = k
∂u ∂u
∂x ∂y ∂y ∂y

∂x ∂y

0
∂v ∂v ∂u ∂v
∂v ∂v
O determinante que aparece no cálculo acima é denominado Jacobiano da transformação
e tem uma notação especial.
96 Meneses, L. R.; Noskoski, O. Cálculo II

Definição 33 Seja T uma transformação do plano uv no plano xy, definida por x =


x(u, v) e y = y(u, v), então o Jacobiano J(T )(u, v) da transformação T é dado por:

∂x ∂x
∂(x, y) ∂u ∂v
∂x ∂y ∂x ∂y

J(T )(u, v) = = = − .
∂(u, v) ∂y ∂y ∂u ∂v ∂v ∂u

∂u ∂v

Considerando a notação acima, podemos obter uma aproximação da área ∆A de R, como



∂(x, y)
∆A ≈
∆u∆v,
∂(u, v)

onde o Jacobiano é calculado em (u0 , v0 ).


Agora, dividindo a região S do plano uv em retângulos Sij e chamando suas imagens no
plano xy de Rij , temos:
Aplicando a aproximação com Jacobiano para ∆A, temos:
Z Z m X
X n
f (x, y)dA ≈ f (xi , yi )∆A
R i=1 j=1
m X n
X ∂(x, y)
≈ f (x(ui , yi ), v(xi , yi )) ∆u∆v
i=1 j=1
∂(u, v)

Note que, a soma dupla é a soma de Riemann para a integral


Z Z
∂(x, y)
f (x(u, y), v(x, y)) ∆u∆v
S ∂(u, v)

Neste texto, iremos considerar a argumentação anterior e enunciar o teorema a seguir.


No entanto, a prova de tal teorema é encontrada em livros de Cálculo Avançado.

Teorema 20 Seja S ⊂ R2 , limitado e com área, e Ω ⊂ R2 um aberto que contém S.


Seja T : Ω → R2 , T (u, v) = (x(u, v), y(u, v)) uma transformação de classe C 1 em Ω,
injetora no interior de S e com J(T )(u, v) 6= 0 para todo (u, v) no interior de S. Nessas
condições, se f = f (x, y) é contı́nua em R = T (S), temos:
Z Z Z Z
f (x, y)dxdy = f (x(u, y), v(x, y))J(T )(u, v)∆u∆v.
R S

Observação 11 Observe que a fórmula de integração em coordenadas polares é um caso


especial do teorema acima. Aqui a transformação T do plano rθ para o plano xy é dada
por
x = x(r, θ) = r cos θ e y = y(r, θ) = rsenθ.

A figura a seguir ilustra a geometria da transformação. Observe que T transforma um


retângulo comum do plano rθ em um retângulo polar do plano xy.
97 Meneses, L. R.; Noskoski, O. Cálculo II

θ y
β θ=β
T θ=β r=b
r=a r=b
α
θ=α r=a
θ=α
a b r x

O Jacobiano de T é dado por



∂x ∂x
∂(x, y) ∂u ∂v
cos θ −rsen θ
= r cos2 θ + rsen2 θ = r > 0.

J(T )(u, v) = = =
∂(u, v) ∂y ∂y sen θ r cos θ

∂u ∂v
Assim,
Z Z Z Z
∂(x, y)
f (x, y)dxdy = f (r cos θ, rsen θ)
dr dθ
R S ∂(r, θ)
Z βZ b
= f (r cos θ, rsen θ)rdrdθ
α a
Z Z
(x + y)9
Exemplo 68 Calcule dxdy, sendo R o domı́nio indicado na figura a seguir.
R y−x
y

−3 −2 −1 0 1 2 3 4 x

Solução: Pela figura acima, observe que, as equações das retas que formam a fronteira do
domı́nio R são x + y = 3, x + y = 4, y − x = 1 e y − x = 3. A ocorrência das expressões
x + y e y − x sugerem que a transformação

u=x+y e v =y−x

poderá ser útil, pois com esta tranformação as linhas de froenteira são curvas de u e v
constantes, correspondem às retas

u = 3, u = 4, v = 1 e v = 3.
98 Meneses, L. R.; Noskoski, O. Cálculo II

y v

T 3

x 3 4 u

Para calcular o Jacobiano da transformação, precisamos determinar a lei da transformação.


Assim, temos:

u = x + y 1 1
⇒ x = (u − v) e y = (u + v),
v = y − x 2 2

e  
1 1
T (u, v) = (x(u, v), y(u, v)) = (u − v), (u + v) .
2 2
Então,
∂x ∂x 1 1
∂(x, y) ∂u − 1
J(T )(u, v) = = ∂v 2 2 = 6= 0
=
∂(u, v) ∂y ∂y 1 1 2

∂u ∂v 2 2
Logo,
Z Z Z Z
(x + y)9 u9 1
dxdy = dudv
R y−x S v 2
Z Z
1 4 3 u9
= dvdu
2 3 1 v
Z
1 4 9
= u [ln v]31 du
2 3
Z 4
1
= ln 3 u9 du
2 3
 10 4
1 u
= ln 3
2 10 3
ln 3 10
= (4 − 310 )
20
99 Meneses, L. R.; Noskoski, O. Cálculo II

Z Z
1 1
Exemplo 69 Calcule sen (x + y) cos (x − y)dxdy, sendo R a região triangular
R 2 2
de vértices A(0, 0), B(2, 0) e C(1, 1).

Solução: A ocorrência das expressões 21 (x + y) e 21 (x − y) no integrando sugere a trans-


formação
1 1
u = (x + y) e v = (x − y).
2 2
Assim, temos:

u = 1 (x + y)
2
⇒ x = u + v e y = u − v.
v = 1 (x − y)
2

Então,
T (u, v) = (x(u, v), y(u, v)) = (u + v, u − v).

y v
T

1 1

2 x 1 u

O Jacobiano da transformação é dado por



∂x ∂x
1 1
J(T )(u, v) = ∂u ∂v =

= −2 6= 0.
∂y ∂y 1 −1

∂u ∂v
Note que, os pontos A(0, 0), B(2, 0) e C(1, 1) são, respectivamente, as imagens de A′ (0, 0),
B ′ (1, 1) e C ′ (1, 0) pela transformação T . De fato, T (0, 0) = (0, 0), T (1, 1) = (2, 0) e
T (1, 0) = (1, 1). Logo,
Z Z Z Z
1 1
sen (x + y) cos (x − y)dxdy = sen u cos v| − 2|dudv
R 2 2 S
Z 1Z u
= 2 sen u cos vdvdu
0 0
Z 1
= 2 sen u [sen v]u0 du
Z0 1
= 2 sen2 u du
Z0 1
1
= 2 (1 − cos 2u)udu
0 2
 1
1 1
= u − sen 2u = 1 − sen 2
2 0 2
100 Meneses, L. R.; Noskoski, O. Cálculo II

RR p
Exemplo 70 Calcule R
x2 + y 2 dxdy, sendo R a região triangular de vértices A(0, 0),
B(1, 0) e C(1, 1).

1
R
1 x

Neste caso, uma mudança de variáveis conveniente seria para coordenadas polares, visto
que, eliminarı́amos o problema da raiz quadrada no integrando. Sabe-se que a trans-
formação de coordenadas polares para coordenadas cartesianas é dada pela lei

T (r, θ) = (x(r, θ), y(r, θ)) = (r cos θ, r senθ),

cujo Jacobiano é dado por J(T )(r, θ) = r.

1
Como x = r cos θ, temos que 1 = r cos θ → r = = sec θ. Assim, para 0 ≤ θ ≤ π4 ,
cos θ
temos 0 ≤ r ≤ sec θ.
Portanto,
Z Z p Z π Z sec θ
4 √
x2 + y 2 dxdy = rrdrdθ
R 0 0
Z π Z sec θ
4
= r2 drdθ
0 0
Z π  3 sec θ
4 r
= dθ
0 3 0
Z π
1 4
= sec3 θdθ
3 0
1 π
= [sec θ · tan θ + ln(sec θ + tan θ)]04
6
1 h√ √ i
= 2 + ln( 2 + 1)
6
R
Observação 12 Calculando-se separadamente a integral sec3 θdθ, obtemos:
Z Z
sec θdθ = sec2 θ · sec θdθ.
3

Note que, esta integral deverá ser calculada por partes, uma vez que temos o produto de
R R
duas funções u · dv, onde u · dv = u · v − v · du.
101 Meneses, L. R.; Noskoski, O. Cálculo II

Fazendo-se u = sec θ e dv = sec2 θdθ, temos du = sec θ · tan θdθ e v = tan θ. Logo,
Z Z
3
sec θdθ = sec θ · tan θ − tan θ · sec θ · tan θdθ
Z
= sec θ · tan θ − tan2 θ · sec θdθ
Z
= sec θ · tan θ − (sec2 θ − 1) · sec θdθ
Z Z
3
= sec θ · tan θ − sec θdθ + sec θdθ
Z Z
3
2 sec θdθ = sec θ · tan θ + sec θdθ
Z
1
sec3 θdθ = [sec θ · tan θ + ln(sec θ + tan θ)]
2

4.11.2 Integral Tripla


Nesta secção será apresentado o método de mudança de variáveis para facilitar o cálculo
de algumas integrais triplas. O método é semelhante ao utilizado em integrais duplas,
exceto pelo fato de que agora trabalharemos com transformações de regiões tridimensio-
nais, em vez de bidimensionais.

Uma mudança de variáveis num subconjunto do R3 é dada por uma transformação

T : R ⊂ R3 → R3
T (u, v, w) = (x(u, v, w), y(u, v, w), z(u, v, w)),

onde R é um subconjunto limitado e com área, T é de classe C 1 e T é injetora.

Definição 34 Se T for a transformação do espaço de variáveis uvw no espaço de


variáveis xyz, definida pelas equações x = x(u, v, w), y(u, v, w) e z(u, v, w), então o
∂(x, y, z)
Jacobiano de T é denotado por J(T )(u, v, w) ou e é definido por
∂(u, v, w)

∂x ∂x ∂x

∂u ∂v ∂w

∂y ∂y ∂y
J(T )(u, v, w) =

∂u ∂v ∂w
∂z ∂z ∂z


∂u ∂v ∂w
102 Meneses, L. R.; Noskoski, O. Cálculo II

Teorema 21 Seja R ⊂ R3 limitado e com área, e Ω ⊂ R3 um aberto que contém S.


Seja T : Ω → R3 , T (u, v, w) = (x(u, v, w), y(u, v, w), z(u, v, w)) uma transformação de
classe C 1 em Ω, injetora no interior de S e com |J(T )(u, v, w)| 6= 0 para todo (u, v, w)
no interior de S. Nessas condições, se f = f (x, y, z) é contı́nua em R = T (S) e temos:
Z Z Z Z Z Z
f (x, y, z)dV = f (x(u, v, w), y(u, v, w), z(u, v, w))|J(T )(u, v, w)|dudvdw
R S

Z Z Z
Exemplo 71 Calcule (z − y)2 xydV , onde R é a região delimitada pelos planos
R
2 4
x = 1, x = 3, z = y, z = y + 1 e os hiperbolóides y = ey= .
x x
Solução: Note que, as equações das superfı́cies que formam o domı́nio R podem ser re-
escritas como x = 1, x = 3, z − y = 0, z − y = 1, xy = 2 e xy = 4. A ocorrência das
expressões x, z − y e xy nas equações de fronteira sugere que a transformação T , definida
por
u = x, v = z − y e w = xy
poderá ser útil, pois com esta transformação as linhas de fronteira são curvas de u, v e w
constantes, correspondentes aos planos

u = 1, u = 3, v = 0, v = 1, w = 2 e w = 4.



 u=x

Resolvendo-se o sistema v = z − y em função de x, y e z, determinamos a lei da



w = xy
transformação T , que é dada por
 w w
T (u, v, w) = u, , v + .
u u
Então, o Jacobiano da transformação é

∂x ∂x ∂x
1 0 0
∂u ∂v ∂w

∂y ∂y ∂y w 1 1
J(T )(u, v, w) = = − 2 0 = −
u u u
∂u ∂v ∂w
∂z ∂z ∂z w 1
− 1

∂u ∂v ∂w
u2 u
Logo,

Z Z Z Z Z Z
2 2
1
(z − y) xydV = v w − dudvdw
R S u
Z 4Z 1Z 3
1
= v 2 w dudvdw
2 0 1 u
103 Meneses, L. R.; Noskoski, O. Cálculo II

Z 4 Z 1
= v 2 w [ln u]31 dvdw
2
Z0 4 Z 1
= ln 3 v 2 wdvdw
2 0
Z 4  3 1
v
= ln 3 w dw
2 3 0
Z
ln 3 4
= wdw
3 2
 4
ln 3 w2
=
3 2 2
= 2 ln 3

4.11.3 Coordenadas Cilı́ndricas e Esféricas


As substituições em coordenadas cilı́ndricas e esféricas são casos especiais do método de
mudança de variáveis em integrais triplas.

Coordenadas Cilı́ndricas

No sistema de coordenadas cilı́ndricas, um ponto P no espaço tridimensional é represen-


tado pela terna ordenada (r, θ, z), onde r e θ são as coordenadas polares da projeção de
P sobre o plano xy e z é a distância direta do plano xy ao ponto P .

P (r, θ, z)

z
r y
θ
x

A lei da transformação T de coordenadas cilı́ndricas (r, θ, z) para coordenadas cartesianas


(x, y, z) é dada por

T (r, θ, z) = (x(r, θ, z), y(r, θ, z), z(r, θ, z)) = (r cos θ, rsen θ, z).
104 Meneses, L. R.; Noskoski, O. Cálculo II

z z
h
h
T

a 2π θ y
a

r x

O Jacobiano J(T )(r, θ, z) é definido por



∂x ∂x ∂x

∂r ∂θ ∂z
cos θ −rsen θ 0
∂y ∂y ∂y
J(T )(r, θ, z) = = sen θ r cos θ 0 = r

∂r ∂θ ∂z
∂z ∂z ∂z 0 0 1


∂r ∂θ ∂z
Assim, uma integral tripla retangular pode ser escrita em coordenadas cilı́ndricas como:
Z Z Z Z Z Z
f (x, y, z)dV = f (r cos θ, rsen θ, z)rdzdrdθ.
R S

Exemplo Z Sabendo que a massa de um sólido pode ser expressa como a integral tripla
Z Z 72
M = ρ(x, y, z)dV , determine a massa do sólido de densidade ρ(x, y, z) = 3 − z,
D p
limitado pelo cone z = x2 + y 2 e pelo plano z = 3.

Solução: A região D é apresentada na figura a seguir.


z
3

y
x

Note que, a equação do cone em coordenadas cilı́ndricas é dada por z = r. Logo, a região
D pode ser expressa em coordenadas cilı́ndricas como

D = {(r, θ, z)/0 ≤ θ ≤ 2π, 0 ≤ r ≤ 3, r ≤ z ≤ 3}.


105 Meneses, L. R.; Noskoski, O. Cálculo II

Assim, temos:
Z Z Z Z Z Z
M = (3 − z)dV = (3 − z)rdzdrdθ
R R
Z 2π Z 3Z 3
= (3 − z)rdzdrdθ
0 0 r
Z 2π Z 3  3
z2
= 3z − rdrdθ
0 0 2 r
Z 2π Z 3   
9 r2
= − 3r − rdrdθ
0 0 2 2

Z 2π Z 3  
9 2 r3
= r − 3r + drdθ
0 0 2 2
Z 2π  2 3
9r 3r3 r4
= − + dθ
0 4 3 8 0
Z
27 2π
= dθ
8 0
27 2π
= [θ]0
8
27π
= unidades de massa.
4
Exemplo 73 Determine o volume do sólido D delimitado pelos parabolóides z = x2 + y 2
e z = 36 − 3x2 − 3y 2 .

Solução: A figura a seguir apresenta um esboço da região de integração.

z
36

y
x

Note, inicialmente, quea curva de interseção dos parabolóides é obtida igualando as coor-
denadas z das equações. Logo,

x2 + y 2 = 36 − 3x2 − 3y 2 → 4x2 + 4y 2 = 36 → x2 + y 2 = 9.

Assim, a projeção sobre o plano xy do sólido delimitado pelos parabolóides é a região

Dxy = {(x, y, 0) ∈ R3 /x2 + y 2 ≤ 9}.


106 Meneses, L. R.; Noskoski, O. Cálculo II

Observe que a região de integração D é facilmente representada em coordenadas cilı́ndricas


por:
D = {(r, θ, z)/0 ≤ θ ≤ 2π, 0 ≤ r ≤ 3 e r2 ≤ z ≤ 36 − 3r2 }.

Logo,

Z Z Z Z Z Z
V = dV = rdzdrdθ
D S
Z 2π Z 3 Z 36−3r2
= rdzdrdθ
0 0 r2
Z 2π Z 3
2
= r [z]r36−3r
2 drdθ
Z0 2π Z0 3
 
= r 36 − 4r2 drdθ
Z0 2π Z0 3
 
= 36r − 4r3 drdθ
Z0 2π 0
 3
= 18r2 − r4 0

0
Z 2π
= 81 dθ
0
= 81 [θ]2π
0 = 162π unidades de volume(u.v.)

Coordenadas Esféricas

Um ponto P (x, y, z) no espaço é representado em coordenadas esféricas por ternas orde-


−→
nadas (ρ, ϕ, θ), sendo ρ a distância de P à origem, ϕ é o ângulo que OP forma com o eixo
−→
OZ positivo (0 ≤ ϕ ≤ π) e θ é o ângulo das coordenadas cilı́ndricas.

ϕ P (ρ, ϕ, θ)
ρ
y
θ
x P′

Note que, considerando o triângulo OP P ′ , retângulo em P ′ , temos r = ρsen ϕ. Como


x = r cos θ e y = r sen θ, podemos escrever x = ρ sen ϕ cos θ e y = ρ sen ϕ sen θ.
107 Meneses, L. R.; Noskoski, O. Cálculo II

ρ ϕ

O r P′

Assim, a lei da transformação T de coordenadas esféricas (ρ, ϕ, θ) para coordenadas car-


tesianas (x, y, z) é dada por

T (ρ, ϕ, θ) = (ρ sen ϕ cos θ, ρ sen ϕ sen θ, ρ cos ϕ).

θ z

T

π y
ϕ

a
ρ x

O Jacobiano J(T )(ρ, ϕ, θ) é dado por:



∂x ∂x ∂x


∂ρ ∂ϕ ∂θ
sen ϕ cos θ ρ cos ϕ cos θ −ρ sen ϕ sen θ


∂y ∂y ∂y

J(T )(ρ, ϕ, θ) = = sen ϕ sen θ ρ cos ϕ sen θ ρ sen ϕ cos θ =
∂ρ ∂ϕ ∂θ

∂z ∂z ∂z

cos ϕ −ρ sen ϕ 0

∂ρ ∂ϕ ∂θ

= ρ2 sen ϕ cos2 ϕ cos2 θ + ρ2 senϕ sen2 ϕ sen2 θ + ρ2 sen ϕ sen2 ϕ cos2 θ + ρ2 sen ϕ cos2 ϕ sen2 θ
= ρ2 sen ϕ sen2 ϕ(sen2 θ + cos2 θ) + ρ2 sen ϕ cos2 ϕ(sen2 θ + cos2 θ)
= ρ2 sen ϕ(sen2 ϕ + cos2 ϕ)
= ρ2 sen ϕ

Logo, uma integral tripla retangular pode ser escrita em coordenadas esféricas como
Z Z Z Z Z Z
f (x, y, z)dV = f (ρ sen ϕ cos θ, ρ sen ϕ sen θ, ρ cos ϕ)ρ2 sen ϕdρdϕdθ
R S
108 Meneses, L. R.; Noskoski, O. Cálculo II

Exemplo 74 Calcule o volume do sólido D limitado acima pela esfera x2 + y 2 + z 2 = 16


p
e abaixo pelo cone 3z = 3x2 + 3y 2 .

Solução: A figura a seguir apresenta um esboço da região D.

Utilizaremos coordenadas esféricas para determinar o volume do sólido em questão. Para


isto, inicialmente determinaremos a equação da esfera e do cone nestas coordenadas.
Assim, temos:

Esfera:

x2 + y 2 + z 2 = 16 ⇒ (ρ sen ϕ cos θ)2 + (ρ sen ϕ sen θ)2 + (ρ cos ϕ)2 = 16


⇒ ρ2 sen2 ϕ cos2 θ + ρ2 sen2 ϕ sen2 θ + ρ2 cos2 ϕ = 16
⇒ ρ2 sen2 ϕ(cos2 θ + sen2 θ) + ρ2 cos2 ϕ = 16
⇒ ρ2 (sen2 ϕ + cos2 ϕ) = 16
⇒ ρ2 = 16
⇒ ρ=4

Cone:
p p
3z = 3x2 + 3y 2 ⇒ 3ρ cos ϕ = 3(ρ sen ϕ cos θ)2 + 3(ρ sen ϕ sen θ)2
p
⇒ 3ρ cos ϕ = 3(ρ2 sen2 ϕ cos2 θ) + 3(ρ2 sen2 ϕ sen2 θ)
p
⇒ 3ρ cos ϕ = 3ρ2 sen2 ϕ(cos2 θ + sen2 θ)

⇒ 3ρ cos ϕ = 3ρ sen ϕ
3 sen ϕ
⇒ √ =
3 cos ϕ
√ sen ϕ
⇒ 3=
cos ϕ
π
⇒ ϕ=
3

Logo, a região D pode ser descrita como


π
Dρ,ϕ,θ = {(ρ, ϕ, θ)/0 ≤ θ ≤ 2π, 0 ≤ ϕ ≤ e 0 ≤ ρ ≤ 4}
3
109 Meneses, L. R.; Noskoski, O. Cálculo II

e o volume dado por

Z Z Z Z Z Z
V = = ρ2 sen ϕdρdϕdθ
Dx,y,z Dρ,ϕ,θ
Z Z π Z
2π 3
4
= ρ2 sen ϕdρdϕdθ
0 0 0
Z 2π Z π  3 4
3 ρ
= sen ϕdϕdθ
0 0 3 0
Z Z π
64 2π 3
= sen ϕdϕdθ
3 0 0
Z Z π
64 2π 3
= [− cos ϕ] dθ
3 0 0
Z
64 2π h π i
= − cos + cos 0 dθ
3 0 3
Z 2π
32 32 2π 64π
= dθ = [θ]0 = u.v
3 0 3 3

Exemplo 75 Calcule o volume do sólido delimitado abaixo pela esfera x2 +y 2 +(z −1)2 =
p
1, acima pelo cone z = x2 + y 2 .

Solução:
Z Z Z
x2 y 2
Exemplo 76 Calculo xdxdydz, onde D é o conjunto + + z 2 ≤ 1.
D 4 9

Solução:

Você também pode gostar